BRS Anatomy - Lower Limb, Lower Limbs, Abdomen and Pelvis, BRS-Abdomen, BRS - Pelvis, 21-30

¡Supera tus tareas y exámenes ahora con Quizwiz!

posteriorly, the perineum end at the

coccyx

the hepatic vein drains into which of the following veins

inferior vena cava

All of the following statements concerning the sciatic nerve the sciatic nerve are correct except

inferior, the nerve lies against the anterior surface of the adductor magnus

the pelvis is closed inferiorly by the

pelvic diaphragm

the pleural cavity, select the incorrect answer

contains lung

a gall stone leaving the gall bladder will pass immediately into the

cycstic duct

which group of structures enters or leaves the porta hepatis

cystic duct, hepatic duct, portal vein

the saphenous nerve is a terminal branch of which of the following nerves

femoral

parts and regions of the lower limb are defined correctly by which of the following

hallux, digit 5

the rectum receives its blodd supply primarily by the branches of

inferior mesenteric arteries

the rectum recieves blood supply primarily by the branches

inferior mesenteric arteries

All of the following statements concerning the ductus deferens are correct except

it begins at the head of the epididymis

all of the following statements concerning the ductus deferns are correct except

it begins at the head of the epididymis

all of the following statements concerning the greater saphenous vein are correct except

it contains no valves

all of the following statements concerning the clitoris are correct except

it contains the female urethra

222. The nerve most commonly injured in fractures of the middle third of the humerus is the A. Ulnar B. Radial C. Median D. Thoracodorsal E. Musculocutaneous

B. Radial

All of the following statements concerning the rectum are correct except

it usually has five transverse folds in its anterior

all of the following statements concerning the vagina are true except

its anterior surface is covered by peritoneum

each of the following veins belong to the portal system except

left suprarenal vein

254. The profunda brachii artery is accompanied by the A. Ulnar nerve B. Radial nerve C. Median nerve D. Long thoracic nerve E. Axillary nerve

B. Radial nerve

a 35 years old man while digging in the yard experienced a dragging sensetion in his right groin. on examination he was found to have a swelling above the medial third of rhe inguinal ligament which expanded on coughing. is it likely to be

an inguinal hernia

all of the following structures empty into the right atrium of the heart except the

anterior cardiac veins

the dorsalis pedis artery is a continuation of the

anterior tibial artery

the four lumbar arteries arise from which of the following vessels

aorta

the ovarian artery is a branch of the following arteries

aorta

the ovarian artery is a branch of which of the following arteries

aorta

the ovarian artery is a branch of which of the following artery

aorta

when the thorax is viewed in PA radiograph, it is impportant to know tata the most superior structure of those listed below is the

aortic arch

the thoracoabdominal intercostal nerves are ventral rami of which of the following nerves

T7-T11

which nerve usually supplies the skin between the big toe and sencond big toe

deep peroneal

the inferior epigastric artery arises from which of the following

external iliac

the inferior epigastric artery is a branch of the

external iliac artery

the abdominal orifice of ht euterine tube is located at its

infundibulum

the thickened, rolled-under portion of the external abdominal oblique aponeurosis is stretched between the anterior superior spine of the ilium and the pubic tubercle, is knows as

inguinal ligament

the thickened, rolled-under portion of the external abdominal oblique aponeurosis which is stretched between the anterior superior spine of the ilium and the pubic tubercle, is knows as

inguinal ligament

the iliolumbar artery is a branch of which of the following arteries

internal iliac

which of the following arteries is the chief blood supply of the perineum

internal pudenal

the pelvic diaphragm is composed of all the following muscles except the

pirifomis

the heaptoduodenal ligament transmits the

portal vein

the hepatoduodenal ligament transmits the

portal vein

which structures are normally concerned with the venous drainage of the stomach

portal vein

Which statement is incorrect about the liver

the hepatic vein carries venous blood from the intestine to the liver

lymphatic drainage of the uterus is directed to all of the following lymph nodes except the

superficial inguinal

the lymphatic drainage of the penis is to which of the following lymph nodes

superficial inguinal

malignancies of the external genitalia and perineal abscesses result in ... of the

superficial inguinal lymph nodes

all of the following structures are traversed by the male urethra except the

superficial transverese perineus muscle

which of the following structures leaves the plevis above the piriform muscle

superior gluteal nerve

the right suprarenal vein drains into which of the following veins

superior mesenteric

which artery is never a branch of the interna liliac artery

superior rectal

the azygos vein drains into

superior vena cava

which statement is incorrect about the liver

the hepatic vein carries venous blood from the intestine to the liver

which of these statements correclty describes the heart

the coronary sinus occupies the posterior interventricular sulcus

all of the following statements about nerves and vessels of the thigh are correct except

the femoral artery terminates in the anterior thigh

each statement belor concerning the femoral triangle is true except

the femoral nerve is covered by the pectinus muscle

Which of the following statements concerning the pericardial sac are correct except

the fibrous pericardium lubricates the moving surfaces of the heart

a blood clot breaking loose and entering the circulation from a leg vein is most likely to lodge and produce ischemia and local infarct in

the lung

All of the following statements concerning the vestibule of the vagina are correct except

the mons pubis is part of the vestibule

all of the following statements concerning the uterus are correct except

uterine artery arises from the external iliac artery

in the female pelvis, peritoneum covers all og the following except the

uterine tube

the broad ligament encloses the

uterine tubes

which of the following structures carries the preganglionic parasympathetic fibers to the liver

vagus nerves

197. Matching questions: A. Olfactory B. Vagus C. Oculomotor D. Abducens E. Facial F. Trigeminal 1. Anosmia 2. Dysphagia 3. Strabismus convergens 4. Bell's palsy 5. The douloreux?

1. A 2. B 3. D 4. E 5. F

198. Matching questions: A. Optic canal B. Superior orbital fissure C. Foramen magnum D. Foramen ovale E. Jugular foramen 1. Opthalmic artery 2. Mandibular nerve 3. Trochlear nerve 4. Glossopharyngeal nerve 5. Vertebral artery 6. Opthalmic nerve

1. A 2. D 3. B 4. E 5. C 6. B

205. Matchin questions: A. Supplementary motor area B. Primary motor area C. Basal ganglia D. Neocerebellum E. Motoneurons alfa and gamma 1. Apraxia 2. Decomposition of movements 3. Tremor 4. Dyskinesia 5. Asynergia 6. Muscular atrophy 7. Flaccid paralysis 8. Sign of Babinski

1. A 2. D 3. C 4. D 5. D 6. E 7. E 8. B

1. The Answer is B. The common peroneal nerve divides into the deep peroneal nerve, which innervates the anterior muscles of the leg and supplies the adjacent skin of the first and second toes, and the superficial peroneal nerve, which innervates the lateral muscles of the leg and supplies the skin on the side of the lower leg and the dorsum of the ankle and foot. The sural nerve supplies the lateral aspect of the foot and the little toe.

1. A 27-year-old patient exhibits a loss of skin sensation and paralysis of muscles on the plantar aspect of the medial side of the foot. Which of the following nerves is most likely damaged? (A) Common peroneal (B) Tibial (C) Superficial peroneal (D) Deep peroneal (E) Sural

203. Matching qustions: A. Visual system B. Vestibular system C. Both D. Neither 1. Receptor of this system belong to mechanoreceptors 2. Subcortical neurons are located in the lateral geniculate body 3. Primary cortical center is in the occipital lobe 4. Fibers reach the flocculonodular lobe 5. Controls sense of balance 6. Does not have connections with motor association cortex

1. B 2. A 3. A 4. C 5. C 6. D

204. Matchin questions: A. Facial nerve B. Glossopharyngeal nerve C. Hypoglossal nerve 1. Supplies hypoglossus muscle 2. Passes throug the jugular foramen 3. Gives off posterior auricular nerve 4. Has motor nucleus in pons

1. C 2. B 3. A 4. A

196. Matching questions: A. Internal carotid B. External carotid C. Vertebral 1. Originates from subclavian artery 2. Gives off facial artery 3. Passes through foramen magnum 4. Gives off anterior cerebral artery 5. Supplies teeth

1. C 2. B 3. C 4. A 5. B

199. Matching questions: A. Pharynx B. Larynx C. Both D. Neither 1. Belongs to the respiratory system 2. Terminates at the level of cricoid cartilage 3. Communicates with the middle ear 4. Is supplied by branches of the external carotid artery 5. Is innervated by branches of the vagus nerve 6. Has cartilages

1. C 2. C 3. A 4. C 5. C 6. B

192. Matching question A. Glossopharyngeal nerve B. Vagus nerve C. Both D. Neither 1. Passes in the parapharyngeal space 2. Innervates muscle of the pharynx 3. Supplies muscles of the larynx 4. Its motor fibers originates in nucleus ambigunas? 5. Leaves the skull through jugular foramen 6. Its parasympathetic fibers originate in the ... 7. Innervates extrinsic muscles of the tragus 8. Gives off nere to carotid sinus

1. C 2. C 3. B 4. C 5. C 6. D 7. D 8. A

195. Matching questions: A. Cranial nerve 3 B. Cranial nerve 5 C. Cranial nerve 7 D. Cranial nerve 10 E. Cranial nerve 11 1. Bell's palsy 2. Loss oflacrimation 3. Hoarseness 4. Deviation of opening jaw to ipsilateral side 5. Absence of light reflexes 6. Facial hemianesthesia 7. Dropping of the upper eyelid (ptosis) 8. Difficulty in swallowing (dysphagia) 9. Weakness in turning head toward opposite side 10. Loss of taste in the anterior two third of tongue

1. C 2. C 3. D 4. B 5. A 6. B 7. A 8. D 9. E 10. C

194. Matching questions: A. Aortic arch 1 B. Aortic arch 2 C. Aortic arch 3 D. Aortic arch 4 E. Aortic arch 5 F. Aortic arch 6 1. Internal carotid artery 2. Arch of the aorta 3. Stapedial artery 4. Maxillary artery 5. Left pulmonary artery 6. Right subclavian artery 7. Common carotid artery

1. C 2. D 3. B 4. A 5. F 6. D 7. C

200. Matching questions: A. Radial nerve B. Ulnar nerve C. Median nerve D. All nerves E. Neither of them 1. Innervates muscles of the forearm 2. Is accompanied by the profunda brachii artery 3. Passes through the lateral (quadrangular) space 4. May be injured during the fracture of the medial epicondyle 5. Innervates biceps brachii muscle 6. Supplies the pronators of the forearm 7. Innervates muscle of the hypothenar 8. Gives of lateral inferior nerve ot the arm

1. D 2. A 3. E 4. B 5. E 6. A 7. B 8. A

202. Matching questions: A. Transverse colon B. Jejenum C. Both D. Neither 1. Develops from the foregut 2. Has circular folds 3. Is an intraperitoneal organ 4. Receives blood from the superior mesentric artery 5. Is innervated by the celiac plexus

1. D 2. B 3. C 4.C 5. D

201. Matching questions: A. Telencephalon B. Diencephalon C. Mesencephalon D. Pons E. Medulla oblongata 1. Pyramidal decussation 2. Decussation lemniscorum 3. Nucleus subthalamicus 4. Dorsal cochlear nucleus 5. Caudate nucleus 6. Medial geniculate body 7. Dorsal vagal nucleus 8. Crus cerebi 9. Mamillary body 10. Lamina tecti (quadrigemina)

1. E 2. E 3. B 4. D 5. A 6. A 7. E 8. C 9. B 10. C

1. A 63-year-old man comes to the emergency department with back pain, weakness, and shortness of breath. On examination, he has an aneurysm of the abdominal aorta at the aortic hiatus of the diaphragm. Which of the following pairs of structures would most likely be compressed? (A) Vagus nerve and azygos vein (B) Esophagus and vagus nerve (C) Azygos vein and thoracic duct (D) Thoracic duct and vagus nerve (E) Inferior vena cava (IVC) and phrenic nerve

1. The answer is C. The aortic hiatus of the diaphragm transmits the azygos vein and thoracic duct. The vagus nerve passes through the esophageal hiatus, and the right phrenic nerve may run through the vena caval hiatus.

1. A 68-year-old woman with uterine carcinoma undergoes surgical resection. This cancer can spread directly to the labia majora in lymphatics that follow which of the following structures? (A) Pubic arcuate ligament (B) Suspensory ligament of the ovary (C) Cardinal (transverse cervical) ligament (D) Suspensory ligament of the clitoris (E) Round ligament of the uterus

1. The answer is E. The round ligament of the uterus runs laterally from the uterus through the deep inguinal ring, inguinal canal, and superfi cial inguinal ring and becomes lost in the subcutaneous tissues of the labium majus. Thus, carcinoma of the uterus can spread directly to the labium majus by traveling in lymphatics that follow the ligament.

10. The Answer is E. The tensor fasciae latae can flex and medially rotate the thigh, so this is the muscle most likely damaged. The hamstring muscles (semitendinosus, semimembran- osus, and biceps femoris) can extend the thigh and flex the leg. The sartorius can flex the thigh and leg. The rectus femoris can flex the thigh and extend the leg. The vastus intermedius can extend the leg.

10. A 20-year-old patient cannot flex and medially rotate the thigh while running and climbing. Which of the following muscles is most likely damaged? (A) Semimembranosus (B) Sartorius (C) Rectus femoris (D) Vastus intermedius (E) Tensor fasciae latae

10. A 27-year-old woman has suffered a gunshot wound to her midabdomen. After examining the patient's angiogram, a trauma surgeon locates the source of bleeding from pairs of veins that typically terminate in the same vein. Which of the following veins are damaged? (A) Left and right ovarian veins (B) Left and right gastroepiploic veins (C) Left and right colic veins (D) Left and right suprarenal veins (E) Left and right hepatic veins

10. The answer is E. The right and left hepatic veins drain into the inferior vena cava (IVC). The right gastroepiploic vein drains into the superior mesenteric vein, but the left one drains into the splenic vein. The right gonadal and suprarenal veins drain into the IVC, whereas the left ones drain into the left renal vein. The right colic vein ends in the superior mesenteric vein, but the left one terminates in the inferior mesenteric vein.

10. A 46-year-old woman has a history of infection in her perineal region. A comprehensive examination reveals a tear of the superior boundary of the superfi cial perineal space. Which of the following structures would most likely be injured? (A) Pelvic diaphragm (B) Colles's fascia (C) Superfi cial perineal fascia (D) Deep perineal fascia (E) Perineal membrane

10. The answer is E. The superior (deep) boundary of the superfi cial perineal space is the perineal membrane (inferior fascia of the urogenital diaphragm). Colles's fascia is the deep membranous layer of the superfi cial perineal fascia. The deep perineal fascia essentially divides the superfi cial perineal space into a superfi cial and deep compartment. The pelvic diaphragm consists of the levator ani and coccygeus muscles.

11. The Answer is C. The groove in the lower surface of the cuboid bone is occupied by the tendon of the peroneus longus muscle. The flexor hallucis longus tendon occupies a groove on the posterior surface of the body of the talus and a groove on the inferior surface of the calcaneus during its course. The tibialis posterior muscle tendon occupies the medial malleolar groove of the tibia. Other muscle tendons are not in the groove of the tarsal bones.

11. A 21-year-old man was involved in a motorcycle accident, resulting in destruction of the groove in the lower surface of the cuboid bone. Which of the following muscle tendons is most likely damaged? (A) Flexor hallucis longus (B) Peroneus brevis (C) Peroneus longus (D) Tibialis anterior (E) Tibialis posterior

11. A 58-year-old man is diagnosed as having a slowly growing tumor in the deep perineal space. Which of the following structures would most likely be injured? (A) Bulbourethral glands (B) Crus of penis (C) Bulb of vestibule (D) Spongy urethra (E) Great vestibular gland

11. The answer is A. The deep perineal space contains the bulbourethral (Cowper's) glands. The crus of the penis, bulb of the vestibule, spongy urethra, and great vestibular gland are found in the superfi cial perineal space.

11. A 43-year-old man complains of abdominal pain just above his umbilicus. On examination, a tumor is found anterior to the IVC. Which of the following structures would most likely be compressed by this tumor? (A) Right sympathetic trunk (B) Left third lumbar artery (C) Third part of the duodenum (D) Left renal artery (E) Cisterna chyli

11. The answer is C. The third part of the duodenum (transverse portion) crosses anterior to the IVC. The other structures do not cross the IVC anteriorly.

12. The Answer is D. The tendon of the flexor hallucis longus muscle occupies first the groove on the posterior surface of the talus and then the groove on the undersurface of the sustentaculum tali. None of the other tendons would have been affected in such an injury.

12. A construction worker falls feet first from a roof. He sustains a fracture of the groove on the undersurface of the sustentaculum tali of the calcaneus bone. Which of the following muscle tendons is most likely torn? (A) Flexor digitorum brevis (B) Flexor digitorum longus (C) Flexor hallucis brevis (D) Flexor hallucis longus (E) Tibialis posterior

12. An elderly man with benign enlargement of his prostate experiences diffi culty in urination, urinary frequency, and urgency. Which of the following lobes of the prostate gland is commonly involved in benign hypertrophy that obstructs the prostatic urethra? (A) Anterior lobe (B) Middle lobe (C) Right lateral lobe (D) Left lateral lobe (E) Posterior lobe

12. The answer is B. The middle lobe of the prostate gland is commonly involved in benign prostatic hypertrophy, resulting in obstruction of the prostatic urethra, whereas the posterior lobe is commonly involved in carcinomatous transformation. The anterior lobe contains little glandular tissue, and the two lateral lobes on either side of the urethra form the major part of the gland.

12. A 33-year-old man with a perforated gastric ulcer complains of excruciating pain in his stomach. It is observed that the pain comes from peritoneal irritation by gastric contents in the lesser sac. Which of the following nerves contain sensory nerve fibers that convey this sharp, stabbing pain? (A) Vagus nerves (B) Greater splanchnic nerves (C) Lower intercostal nerves (D) White rami communicantes (E) Gray rami communicantes

12. The answer is C. Pain sensation originating from peritoneal irritation by gastric contents in the lesser sac is carried by lower intercostals nerves. The vagus nerves carry sensory fibers associated with reflexes in the gastrointestinal (GI) tract. The greater splanchnic nerves and white rami communicantes carry pain (general visceral afferent [GVA]) fibers from the wall of the stomach and other areas of the GI tract. The gray rami communicantes contains no sensory fibers but contain sympathetic postganglionic fibers.

13. The Answer is D. The greater saphenous vein ascends superficial to the fascia lata. It courses anterior to the medial malleolus and posterior to the medial condyles of the tibia and femur and terminates in the femoral vein by passing through the saphenous opening. The small saphenous vein drains into the popliteal vein. The greater saphenous vein does not run along with the femoral artery.

13. A thoracic surgeon is going to collect a portion of the greater saphenous vein for coronary bypass surgery. He has observed that this vein runs: (A) Posterior to the medial malleolus (B) Into the popliteal vein (C) Anterior to the medial condyles of the tibia and femur (D) Superficial to the fascia lata of the thigh (E) Along with the femoral artery

13. A young boy is brought to the hospital after a bicycle accident and possible pelvic fracture. While awaiting a computed tomography (CT) scan of his pelvis, a physician proceeds with a focal neurologic examination. In testing the child's reflexes, which of the following nerves would carry afferent impulses of the cremasteric reflex? (A) Subcostal nerve (B) Lateral femoral cutaneous nerve (C) Genitofemoral nerve (D) Iliohypogastric nerve (E) Femoral nerve

13. The answer is C. Stimulation of the cremaster muscle draws the testis up from the scrotum toward the superficial inguinal ring. The efferent limb of the reflex arc is the genital branch of the genitofemoral nerve, whereas the afferent limb is the femoral branch of the genitofemoral nerve. The other nerves are not involved in the cremasteric reflex.

13. A 59-year-old man is diagnosed with prostate cancer following a digital rectal examination. For the resection of prostatecancer, it is important to know that the prostatic ducts open into or on which of the following structures? (A) Membranous part of the urethra (B) Seminal colliculus (C) Spongy urethra (D) Prostatic sinus (E) Prostatic utricle

13. The answer is D. Ducts from the prostate gland open into the prostatic sinus, which is a groove on either side of the urethral crest. The prostate gland receives the ejaculatory duct, which opens into the prostatic urethra on the seminal colliculus (a prominent elevation of the urethral crest) just lateral to the prostatic utricle, which is a small blind pouch. The bulbourethral gland lies on the lateral side of the membranous urethra within the deep perineal space, but its duct opens into the bulbous portion of the spongy (penile) urethra.

14. The Answer is D. The quadriceps femoris muscle includes the rectus femoris muscle and the vastus medialis, intermedialis, and lateralis muscles. They extend the leg at the knee joint. The semitendinosus, semimembranosus, and biceps femoris muscles (the hamstrings) extend the thigh and flex the leg. The sartorius and gracilis muscles can flex the thigh and the leg.

14. A 52-year-old woman slipped and fell and now complains of being unable to extend her leg at the knee joint. Which of the following muscles was paralyzed as a result of this accident? (A) Semitendinosus (B) Sartorius (C) Gracilis (D) Quadriceps femoris (E) Biceps femoris

14. A 29-year-old woman with a ruptured ectopic pregnancy is admitted to a hospital for culdocentesis. A long needle on the syringe is most effi ciently inserted through which of the following structures? (A) Anterior fornix of the vagina (B) Posterior fornix of the vagina (C) Anterior wall of the rectum (D) Posterior wall of the uterine body (E) Posterior wall of the bladder

14. The answer is B. A needle should be inserted through the posterior fornix just below the posterior lip of the cervix while the patient is in the supine position to aspirate abnormal fl uid in the cul-de-sac of Douglas (rectouterine pouch). Rectouterine excavation is not most effi ciently aspirated by puncture of other structures.

14. A 21-year-old man receives a penetrating knife wound in the abdomen and is injured in both the superior mesenteric artery and the vagus nerve. Which portion of the colon would most likely be impaired by this injury? (A) Ascending and descending colons (B) Transverse and sigmoid colons (C) Descending and sigmoid colons (D) Ascending and transverse colons (E) Transverse and descending colons

14. The answer is D. The ascending and transverse colons receive blood from the superior mesenteric artery and parasympathetic nerve fibers from the vagus nerve. However, the descending and sigmoid colons receive blood from the inferior mesenteric artery and the parasympathetic nerve fibers from the pelvic splanchnic nerve arising from sacral spinal nerves (S2-S4).

15. The Answer is C. The tibialis anterior can dorsiflex and invert the foot. The peroneus lon- gus and brevis muscles can plantar flex and evert the foot, the peroneus tertius can dorsiflex and evert the foot, and the extensor digitorum longus can dorsiflex the foot and extend the toes.

15. A patient experiences weakness in dorsiflexing and inverting the foot. Which of the following muscles is damaged? (A) Peroneus longus (B) Peroneus brevis (C) Tibialis anterior (D) Extensor digitorum longus (E) Peroneus tertius

15. A 37-year-old man is suffering from carcinoma of the skin of the penis. Cancer cells are likely to metastasize directly to which of the following lymph nodes? (A) External iliac nodes (B) Internal iliac nodes (C) Superfi cial inguinal nodes (D) Aortic (lumbar) nodes (E) Deep inguinal nodes

15. The answer is C. The superfi cial inguinal nodes receive lymph from the penis, scrotum, buttocks, labium majus, and the lower parts of the vagina and anal canal. These nodes have efferent vessels that drain primarily into the external iliac and common iliac nodes and ultimately to the lumbar (aortic) nodes. The internal iliac nodes receive lymph from the upper part of the rectum, vagina, uterus, and other pelvic organs, and they drain into the common iliac nodes and then into the lumbar (aortic) nodes. Lymph vessels from the glans penis drain initially into the deep inguinal nodes and then into the external iliac nodes.

15. A 42-year-old man with portal hypertension secondary to cirrhosis of the liver and subsequent massive ascites presents to the emergency department. He refuses to have a transjugular intrahepatic portosystemic shunt (TIPS) procedure and prefers surgery. Which of the following surgical connections is involved in the most practical method of shunting portal blood around the liver? (A) Superior mesenteric vein to the inferior mesenteric vein (B) Portal vein to the superior vena cava (C) Portal vein to the left renal vein (D) Splenic vein to the left renal vein (E) Superior rectal vein to the left colic vein

15. The answer is D. Portal hypertension can be reduced by diverting blood from the portal to the caval system. This is accomplished by connecting the splenic vein to the left renal vein or by creating a communication between the portal vein and the IVC. A connection between a hepatic vein and a branch of the portal vein can be accomplished by the transjugular intrahepatic portosystemic shunt (TIPS) procedure in the treatment of bleeding esophageal varices.

16. The Answer is C. The obturator artery gives rise to an acetabular branch that runs in the round ligament of the head of the femur.

16. Rupture of the ligamentum teres capitis femoris may lead to damage to a branch of which of the following arteries? (A) Medial circumflex femoral (B) Lateral circumflex femoral (C) Obturator (D) Superior gluteal (E) Inferior gluteal

16. A 42-year-old woman who has had six children develops a weakness of the urogenital diaphragm. Paralysis of which of the following muscles would cause such a symptom? (A) Sphincter urethrae (B) Coccygeus (C) Superfi cial transversus perinei (D) Levator ani (E) Obturator internus

16. The answer is A. The urogenital diaphragm consists of the sphincter urethrae and deep transverse perineal muscles. Weakness of the muscles, ligaments, and fasciae of the pelvic fl oor, such as the pelvic diaphragm, urogenital diaphragm, and cardinal (transverse cervical) ligaments, occurs as result of multiple child delivery, advancing age, and menopause. The pelvic diaphragm is composed of the levator ani and coccygeus muscles. The superfi - cial transversus perinei is one of the superfi cial perineal muscles, and the obturator internus forms the lateral wall of the ischiorectal fossa.

16. A 78-year-old man is suffering from ischemia of the suprarenal glands. This condition results from rapid occlusion of direct branches of which of the following arteries? (A) Aorta, splenic, and inferior phrenic arteries (B) Renal, splenic, and inferior mesenteric arteries (C) Aorta, inferior phrenic, and renal arteries (D) Superior mesenteric, inferior mesenteric, and renal arteries (E) Aorta and hepatic and renal arteries

16. The answer is C. The suprarenal gland receives arteries from three sources. The superior suprarenal artery arises from the inferior phrenic artery, the middle suprarenal artery arises from the abdominal aorta, and the inferior suprarenal artery arises from the renal artery. The hepatic, superior mesenteric, inferior mesenteric, and splenic arteries do not supply the suprarenal gland.

17. The Answer is D. In adults, the chief arterial supply to the head of the femur is from the branches of the medial femoral circumflex artery. The lateral femoral circumflex artery may supply the femoral head by anastomosing with the medial femoral circumflex artery. The posterior branch of the obturator artery gives rise to the artery of the head of the femur, which runs in the round ligament of the femoral head and is usually insufficient to supply the head of the femur in adults but is an important source of blood to the femoral head in children. The superior and inferior gluteal arteries do not supply the head of the femur.

17. Fracture of the neck of the femur results in avascular necrosis of the femoral head, probably resulting from lack of blood supply from which of the following arteries? (A) Obturator (B) Superior gluteal (C) Inferior gluteal (D) Medial femoral circumflex (E) Lateral femoral circumflex

17. A 43-year-old man has a benign tumor located near a gap between the arcuate pubic ligament and the transverse perineal ligament. Which of the following structures is most likely compressed by this tumor? (A) Perineal nerve (B) Deep dorsal vein of the penis (C) Superfi cial dorsal vein (D) Posterior scrotal nerve (E) Deep artery of the penis

17. The answer is B. The deep dorsal vein, dorsal artery, and dorsal nerve of the penis pass through a gap between the arcuate pubic ligament and the transverse perineal ligament. The perineal nerve divides into a deep branch, which supplies all of the perineal muscles, and superfi cial branches as posterior scrotal nerves that supply the scrotum. The superfi - cial dorsal vein of the penis empties into the greater saphenous vein. The deep artery of the penis runs in the corpus cavernosum of the penis

17. A radiograph of a 32-year-old woman reveals a perforation in the posterior wall of the stomach in which the gastric contents have spilled into the lesser sac. The general surgeon has opened the lienogastric (gastrosplenic) ligament to reach the lesser sac and notes erosion of the ulcer into an artery. Which of the following vessels is most likely involved? (A) Splenic artery (B) Gastroduodenal artery (C) Left gastric artery (D) Right gastric artery (E) Left gastroepiploic artery

17. The answer is E. The left gastroepiploic artery runs through the lienogastric ligament, and hence, it is the artery most likely injured. The splenic artery is found in the lienorenal ligament. The right and left gastric arteries run within the lesser omentum. The gastroduodenal artery descends between the duodenum and the head of the pancreas.

18. The Answer is C. The acetabulum is a cup-shaped cavity on the lateral side of the hip bone and is formed superiorly by the ilium, posteroinferiorly by the ischium, and anteromedially by the pubis. The sacrum and the head of the femur do not participate in the formation of the acetabulum.

18. If the acetabulum is fractured at its posterosuperior margin by dislocation of the hip joint, which of the following bones could be involved? (A) Pubis (B) Ischium (C) Ilium (D) Sacrum (E) Head of the femur

18. A 35-year-old woman with a history of cholecystectomy arrives in the emergency department with intractable hiccups most likely caused by an abdominal abscess secondary to surgical infection. Which of the following nerves carries pain sensation caused by irritation of the peritoneum on the central portion of the inferior surface of the diaphragm? (A) Vagus nerve (B) Lower intercostal nerve (C) Phrenic nerve (D) Greater splanchnic nerve (E) Subcostal nerve

18. The answer is C. The diaphragm receives somatic motor fibers solely from the phrenic nerves. However, the peritoneum on the central part of the diaphragm receives sensory fibers from the phrenic nerve, and the peripheral part of the diaphragm receives such fibers from the lower intercostal nerves. The subcostal nerve supplies the peritoneum inferior to the diaphragm. The vagus and greater splanchnic nerves do not carry pain fibers from the peritoneum.

18. An obstetrician performs a median episiotomy on a woman before parturition to prevent uncontrolled tearing. If the perinealbody is damaged, the function of which of the following muscles might be impaired? (A) Ischiocavernosus and sphincter urethrae (B) Deep transverse perineal and obturator internus (C) Bulbospongiosus and superfi cial transverse perineal (D) External anal sphincter and sphincter urethrae (E) Bulbospongiosus and ischiocavernosus

18. The answer is C. The perineal body (central tendon of the perineum) is a fi bromuscular node at the center of the perineum. It provides attachment for the bulbospongiosus, the superfi cial and deep transverse perineal muscles, and the sphincter ani externus muscles. Other muscles (ischiocavernosus, sphincter urethrae, and obturator internus) are not attached to the perineal body

19. The Answer is E. The gluteus medius or minimus abducts and rotates the thigh medially. The piriformis, obturator internus, quadratus femoris, and gluteus maximus muscles can rotate the thigh laterally.

19. A woman experiences weakness when abducting and medially rotating the thigh after this accident. Which of the following muscles is most likely damaged? (A) Piriformis (B) Obturator internus (C) Quadratus femoris (D) Gluteus maximus (E) Gluteus minimus

19. A 22-year-old man has a gonorrheal infection that has infi ltrated the space between the inferior fascia of the urogenital diaphragm and the superfi cial perineal fascia. Which of the following structures might be infl amed? (A) Bulb of the penis (B) Bulbourethral gland (C) Membranous part of the male urethra (D) Deep transverse perineal muscle (E) Sphincter urethrae

19. The answer is A. The bulb of the penis is located in the superfi cial perineal space between the inferior fascia of the urogenital diaphragm and the membranous layer of the superfi cial perineal fascia (Colles's fascia). All of the other structures are found in the deep perineal pouch.

19. A 16-year-old boy with a ruptured spleen comes to the emergency department for splenectomy. Soon after ligation of the splenic artery just distal to its origin, a surgical resident observes that the patient is healing normally. Normal blood flow would occur in which of the following arteries? (A) Short gastric arteries (B) Dorsal pancreatic artery (C) Inferior pancreaticoduodenal artery (D) Left gastroepiploic artery (E) Artery in the lienorenal ligament

19. The answer is C. The inferior pancreaticoduodenal artery is a branch of the superior mesenteric artery. All of other arteries are branches of the splenic artery.

2. The Answer is C. The superior gluteal nerve innervates the gluteus medius muscle. Paralysis of this muscle causes gluteal gait, a waddling gait characterized by a falling of the pelvis toward the unaffected side at each step. The gluteus medius muscle normally functions to stabilize the pelvis when the opposite foot is off the ground. The inferior gluteal nerve inner- vates the gluteus maximus, and the nerve to the obturator internus supplies the obturator internus and superior gemellus muscles. The obturator nerve innervates the adductor mus-cles of the thigh, and the femoral nerve supplies the flexors of the thigh.

2. A patient with a deep knife wound in the buttock walks with a waddling gait that is characterized by the pelvis falling toward one side at each step. Which of the following nerves is damaged? (A) Obturator nerve (B) Nerve to obturator internus (C) Superior gluteal nerve (D) Inferior gluteal nerve (E) Femoralnerve

2. A 17-year-old boy suffers a traumatic groin injury during a soccer match. The urologist notices tenderness and swelling of the boy's left testicle that may be produced by thrombosis in which of the following veins? (A) Left internal pudendal vein (B) Left renal vein (C) Inferior vena cava (D) Left inferior epigastric vein (E) Left external pudendal vein

2. The answer is B. A tender swollen left testis may be produced by thrombosis in the left renal vein because the left testicular vein drains into the left renal vein. The right testicular vein drains into the inferior vena cava. The left internal pudendal vein empties into the left internal iliac vein. The left inferior epigastric vein drains into the left external iliac vein, and the left external pudendal vein empties into the femoral vein.

2. A 36-year-old woman with yellow pigmentation of the skin and sclerae presents at the outpatient clinic. Which of the following conditions most likely is the cause of her obstructive jaundice? (A) Aneurysm of the splenic artery (B) Perforated ulcer of the stomach (C) Obstruction of the main pancreatic duct (D) Cancer in the head of the pancreas (E) Cancer in the body of the pancreas

2. The answer is D. Because the bile duct traverses the head of the pancreas, cancer in the head of the pancreas obstructs the bile duct, resulting in jaundice. Aneurysm of the splenic artery, obstruction of the main pancreatic duct, a stomach ulcer, and cancer in the body of the pancreas are not closely associated with the bile duct. The tail of the pancreas is located at the hilus of the spleen, which lies far from the bile duct.

20. The Answer is C. The superior gluteal artery does not participate in the cruciate anasto- mosis of the thigh. The inferior gluteal artery, transverse branches of the medial and lat- eral femoral circumflex arteries, and an ascending branch of the first perforating artery form the cruciate anastomosis of the thigh.

20. If all of the arteries that are part of the cruciate anastomosis of the upper thigh are ligated, which of the following arteries maintains blood flow? (A) Medial femoral circumflex (B) Lateral femoral circumflex (C) Superior gluteal (D) Inferior gluteal (E) First perforating

20. A 9-year-old boy was admitted to the emergency department complaining of nausea, vomiting, fever, and loss of appetite. On examination, he was found to have tenderness and pain on the right lower quadrant. Based on signs and symptoms, the diagnosis of acute appendicitis was made. During an appendectomy performed at McBurney point, which of the following structures is most likely to be injured? (A) Deep circumflex femoral artery (B) Inferior epigastric artery (C) Iliohypogastric nerve (D) Genitofemoral nerve (E) Spermatic cord

20. The answer is C. The iliohypogastric nerve runs medially and inferiorly between the internal oblique and transverse abdominal muscles near the McBurney point, the point at the junction of the lateral one-third of the line between the anterior superior iliac spine and the umbilicus. Other structures are not found near the McBurney point.

20. A 39-year-old man is unable to expel the last drops of urine from the urethra at the end of micturition because of paralysis of the external urethral sphincter and bulbospongiosus muscles. This condition may occur as a result of injury to which of the following nervous structures? (A) Pelvic plexus (B) Prostatic plexus (C) Pudendal nerve (D) Pelvic splanchnic nerve (E) Sacral splanchnic nerve

20. The answer is C. The perineal branch of the pudendal nerve supplies the external urethral sphincter and bulbospongiosus muscles in the male. All other nervous structures do not supply skeletal muscles but supply smooth muscles in the perineal and pelvic organs. The pelvic and prostatic plexuses contain both sympathetic and parasympathetic nerve fi bersThe pelvic splanchnic nerve carries preganglionic parasympathetic fi bers, whereas the sacral splanchnic nerve transmits preganglionic sympathetic fi bers.

21. The Answer is C. Reduction of blood flow in the medial tarsal artery occurs because it is a branch of the dorsalis pedis artery, which begins at the ankle joint as the continuation of the anterior tibial artery. The anterior tibial and peroneal arteries supply the peroneus longus muscle. The deep plantar arterial arch is formed mainly by the lateral plantar artery. Blood pressure in the anterior tibial artery should be higher than normal. The arcuate artery should have a low blood pressure because it is a terminal branch of the dorsalis pedis artery.

21. A 34-year-old woman sustains a deep cut on the dorsum of the foot just distal to her ankle joint by a falling kitchen knife. A physician in the emergency department has ligated the dorsalis pedis artery proximal to the injured area. Which of the following con- ditions most likely occurs as a result of the injury? (A) Ischemia in the peroneus longus muscle (B) Aneurysm in the plantar arterial arch (C) Reduction of blood flow in the medial tarsal artery (D) Low blood pressure in the anterior tibial artery (E) High blood pressure in the arcuate artery

21. A 21-year-old marine biologist asks about her fi rst bimanual examination, and it is explained to her that the normal position of the uterus is: (A) Antefl exed and anteverted (B) Retrofl exed and anteverted (C) Antefl exed and retroverted (D) Retroverted and retrofl exed (E) Anteverted and retroverted

21. The answer is A. The normal position of the uterus is anteverted (i.e., angle of 90 degrees at the junction of the vagina and cervical canal) and antefl exed (i.e., angle of 160 to 170 degrees at the junction of the cervix and body).

24. A 23-year-old massage therapist who specializes in women's health attends a lecture at an annual conference on techniques of massage. She asks, "What structure is drained by the lumbar (aortic) lymph nodes?" Which of the following structures is the correct answer to this question? (A) Perineum (B) Lower part of the vagina (C) External genitalia (D) Ovary (E) Lower part of the anterior abdominal wall

24. The answer is D. The lymphatic vessels from the ovary ascend with the ovarian vessels in the suspensory ligament and terminate in the lumbar (aortic) nodes. Lymphatic vessels from the perineum, external genitalia, and lower part of the anterior abdominal wall drain into the superfi cial inguinal nodes

21. A 54-year-old man with a long history of alcohol abuse presents to the emergency department with rapidly increasing abdominal distention most likely resulting from an alteration in portal systemic blood flow. Which of the following characteristics is associated with the portal vein or the portal venous system? (A) Lower blood pressure than in the IVC (B) Least risk of venous varices because of portal hypertension (C) Distention of the portal vein resulting from its numerous valves (D) Caput medusae and hemorrhoids caused by portal hypertension (E) Less blood flow than in the hepatic artery

21. The answer is D. Portal hypertension can cause esophageal varices, caput medusa, and hemorrhoids. The portal vein has higher pressure than systemic veins; the vein and its tributaries have no valves, or, if present, they are insignificant. In addition, the portal vein carries two to three times as much blood as the hepatic artery.

22. The Answer is A. The vastus lateralis muscles arise from the femur and all the other muscles originate from the hip (coxal) bone. The biceps femoris inserts on the fibula, and other muscles insert on the tibia; thus, all of them contribute to the stability of the knee joint.

22. A patient experiences paralysis of the muscle that originates from the femur and contributes directly to the stability of the knee joint. Which of the following muscles is involved? (A) Vastus lateralis (B) Semimembranosus (C) Sartorius (D) Biceps femoris (long head) (E) Rectus femoris

22. After his bath but before getting dressed, a 4-year-old boy was playing with his puppy. The boy's penis was bitten by the puppy, and the deep dorsal vein was injured. The damaged vein: (A) Lies superfi cial to Buck's fascia (B) Drains into the prostatic venous plexus (C) Lies lateral to the dorsal artery of the penis (D) Is found in the corpus spongiosum (E) Is dilated during erection

22. The answer is B. The deep dorsal vein of the penis lies medial to the dorsal artery of the penis on the dorsum of the penis and deep to Buck's fascia, drains into the prostatic plexus of veins, and is compressed against the underlying deep fascia of the penis during erection.

22. While examining radiographs and angiograms of a 52-year-old patient, a physician is trying to distinguish the jejunum from the ileum. He has observed that the jejunum has: (A) Fewer plicae circulares (B) Fewer mesenteric arterial arcades (C) Less digestion and absorption of nutrients (D) Shorter vasa recta (E) More fat in its mesentery

22. The answer is B. The jejunum has fewer mesenteric arterial arcades but longer vasa recta than the ileum. The plicae circulares (circular folds) are tall and closely packed in the jejunum and are low and sparse in the ileum, and the lower part of the ileum has no plicae circulares. More digestion and absorption of nutrients occurs in the jejunum than in the ileum, and less fat is found in the mesentery of the jejunum.

23. The Answer is E. The superficial peroneal nerve emerges between the peroneus longus and peroneus brevis muscles and descends superficial to the extensor retinaculum of the ankle on the anterolateral side of the leg and ankle, innervating the skin of the lower leg and foot. The great saphenous vein begins at the medial end of the dorsal venous arch of the foot and ascends in front of the medial malleolus and along the medial side of the tibia along with the saphenous nerve. Other structures pass deep to the extensor retinaculum.

23. A patient is involved in a motorcycle wreck that results in avulsion of the skin over the anterolateral leg and ankle. Which of the following structures is most likely destroyed with this type of injury? (A) Deep peroneal nerve (B) Extensor digitorum longus muscle tendon (C) Dorsalis pedis artery (D) Great saphenous vein (E) Superficialperonealnerve

23. A 67-year-old woman with a long history of liver cirrhosis was seen in the emergency department. In this patient with portal hypertension, which of the following veins is most likely to be dilated? (A) Right colic vein (B) Inferior epigastric vein (C) Inferior phrenic vein (D) Suprarenal vein (E) Ovarian vein

23. The answer is A. The right colic vein belongs to the portal venous system and empties into the superior mesenteric vein, which joins the splenic vein to form the portal vein. The inferior epigastric, inferior phrenic, suprarenal, and ovarian veins belong to the systemic (or caval) venous system and drain directly or indirectly into the IVC.

23. A 62-year-old man is incapable of penile erection after rectal surgery with prostatectomy. The patient most likely has a lesion of which of the following nerves? (A) Dorsal nerve of the penis (B) Perineal nerve (C) Hypogastric nerve (D) Sacral splanchnic nerve (E) Pelvic splanchnic nerve

23. The answer is E. The pelvic splanchnic nerve contains preganglionic parasympathetic fi bers, whereas the sacral splanchnic nerve contains preganglionic sympathetic fi bers. Parasympathetic fi bers are responsible for erection, whereas sympathetic fi bers are involved with ejaculation. The right and left hypogastric nerves contain primarily sympathetic fi bers and visceral sensory fi bers. The dorsal nerve of the penis and the perineal nerve provide sensory nerve fi bers.

24. The Answer is E. The lesser (small) saphenous vein ascends on the back of the leg in com- pany with the sural nerve and terminates in the popliteal vein. The peroneal vein empties into the posterior tibial vein. The anterior and posterior tibial veins are deep veins and join to form the popliteal vein. The great saphenous vein drains into the femoral vein.

24. A knife wound penetrates the superficial vein that terminates in the popliteal vein. Bleeding occurs from which of the following vessels? (A) Posterior tibial vein (B) Anterior tibial vein (C) Peroneal vein (D) Great saphenous vein (E) Lessersaphenousvein

24. A 26-year-old patient is admitted to a local hospital with a retroperitoneal infection. Which of the following arteries is most likely to be infected? (A) Left gastric artery (B) Proper hepatic artery (C) Middle colic artery (D) Sigmoid arteries (E) Dorsal pancreatic artery

24. The answer is E. The pancreas is a retroperitoneal organ, except for a small portion of its tail. The dorsal pancreatic artery would be the infected artery because it arises from the splenic artery and runs retroperitoneally along the superior border of the pancreas behind the peritoneum. The other arteries run within layers of the peritoneum. The left gastric arteries run within the lesser omentum; the proper hepatic artery runs within the free margin of the lesser omentum; the middle colic artery runs within the transverse mesocolon; the sigmoid arteries run within the sigmoid mesocolon.

25. The Answer is B. The keystone of the medial longitudinal arch of the foot is the head of the talus, which is located at the summit between the sustentaculum tali and the navicular bone. The medial longitudinal arch is supported by the spring ligament and the tendon of the flexor hallucis longus muscle. The cuboid bone serves as the keystone of the lateral longitudinal arch, which is supported by the peroneus longus tendon and the long and short plantar ligaments. The transverse arch is formed by the navicular, three cuneiform, the cuboid, and five metatarsal bones and is supported by the peroneus longus tendon and the transverse head of the adductor hallucis.

25. A 10-year-old boy falls from a tree house. The resultant heavy compression of the sole of his foot against the ground caused a fracture of the head of the talus. Which of the following structures is unable to function normally? (A) Transverse arch (B) Medial longitudinal arch (C) Lateral longitudinal arch (D) Tendon of the peroneus longus (E) Long plantar ligament

25. A pediatric surgeon has resected a structure that is a fibrous remnant of an embryonic or fetal artery in a 5-year-old child. Which of the following structures is most likely to be divided? (A) Lateral umbilical fold (B) Medial umbilical fold (C) Median umbilical fold (D) Ligamentum teres hepatis (E) Ligamentum venosum

25. The answer is B. The medial umbilical fold or ligament contains a fibrous remnant of the umbilical artery. The median umbilical fold contains a fibrous remnant of the urachus. The lateral umbilical fold (ligament) contains the inferior epigastric artery and vein, which are adult blood vessels. The ligamentum venosum contains a fibrous remnant of the ductus venosus, and the ligamentum teres hepatic contains a fibrous remnant of the left umbilical vein.

25. A sexually active adolescent presents with an infection within the ischiorectal fossa. Which of the following structures is most likely injured? (A) Vestibular bulb (B) Seminal vesicle (C) Greater vestibular gland (D) Inferior rectal nerve (E) Internal pudendal artery

25. The answer is D. The ischiorectal fossa contains the inferior rectal nerves and vessels and adipose tissue. The bulb of the vestibule and the great vestibular gland are located in the superfi cial perineal space, whereas the bulbourethral gland is found in the deep perineal space. The internal pudendal artery runs in the pudendal canal, but its branches pass through the superfi cial and deep perineal spaces.

26. The Answer is D. The gluteus maximus can extend and rotate the thigh laterally. The obturator externus rotates the thigh laterally. The sartorius can flex both the hip and knee joints. The tensor fasciae latae can flex and medially rotate the thigh. The semitendinosus can extend the thigh and medially rotate the leg.

26. A 24-year-old woman complains of weakness when she extends her thigh and rotates it laterally. Which of the following muscles is paralyzed? (A) Obturator externus (B) Sartorius (C) Tensor fasciae latae (D) Gluteus maximus (E) Semitendinosus

3. On a busy Saturday night in Chicago, a 16-year-old boy presents to the emergency department with a stab wound from a knife that entered the pelvis above the piriformis muscle. Which of the following structures is most likely to be damaged? (A) Sciatic nerve (B) Internal pudendal artery (C) Superior gluteal nerve (D) Inferior gluteal artery (E) Posterior femoral cutaneous nerve

3. The answer is C. The superior gluteal nerve leaves the pelvis through the greater sciatic foramen, above the piriformis. The sciatic nerve, internal pudendal vessels, inferior gluteal vessels and nerve, and posterior femoral cutaneous nerve leave the pelvis below the piriformis

26. A fi rst-year resident in the urology department reviews pelvic anatomy before seeing patients. Which of the following statements is correct? (A) The dorsal artery of the penis supplies the glans penis. (B) The seminal vesicles store spermatozoa. (C) The duct of the bulbourethral gland opens into the membranous urethra. (D) The duct of the greater vestibular gland opens into the vagina. (E) The anterior lobe of the prostate gland is prone to carcinomatous transformation.

26. The answer is A. The dorsal artery of the penis supplies the glans penis. The seminal vesicles store no spermatozoa. The duct of the bulbourethral gland opens into the bulbous portion of the spongy urethra, whereas the greater vestibular gland opens into the vestibule between the labium minora and the hymen. The anterior lobe of the prostate is devoid of glandular substance, the middle lobe is prone to benign hypertrophy, and the posterior lobe is prone to carcinomatous transformation

26. A 57-year-old patient has a tumor in the body of the pancreas that obstructs the inferior mesenteric vein just before joining the splenic vein. Which of the following veins is most likely to be enlarged? (A) Middle colic vein (B) Left gastroepiploic vein (C) Inferior pancreaticoduodenal vein (D) Ileocolic vein (E) Left colic vein

26. The answer is E. The left colic vein is a tributary of the inferior mesenteric vein. The middle colic, inferior pancreaticoduodenal, and ileocolic veins drain into the superior mesenteric vein. The left gastroepiploic vein empties into the splenic vein.

27. The Answer is D. If the proximal end of the popliteal artery is blocked, blood may reach the foot by way of the descending branch of the lateral circumflex femoral artery, which participates in the anastomosis around the knee joint. Other blood vessels are direct or indirect branches of the popliteal artery.

27. A patient with hereditary blood clotting problems presents with pain in the back of her knee. An arteriogram reveals a blood clot in the popliteal artery at its proximal end. Which of the following arteries will allow blood to reach the foot? (A) Anterior tibial (B) Posterior tibial (C) Peroneal (D) Lateral circumflex femoral (E) Superior medial genicular

27. A 43-year-old woman presents with a prolapsed uterus. Repair of a prolapsed uterus requires knowledge of the supporting structures of the uterus. Which of the following structures plays the most important role in the support of the uterus? (A) Levator ani (B) Sphincter urethrae (C) Uterosacral ligament (D) Ovarian ligament (E) Arcuate pubic ligament

27. The answer is A. The pelvic diaphragm, particularly the levator ani, provides the most important support for the uterus, although the urogenital diaphragm and the uterosacral and ovarian ligaments support the uterus. The arcuate pubic ligament arches across the inferior aspect of the pubic symphysis.

27. An elderly man with prostatic hypertrophy returns to his urologist with another case of epididymitis. An acute infection involving the dartos muscle layer of the scrotum most likely leads to an enlargement of which of the following lymph nodes? (A) Preaortic nodes (B) Lumbar nodes (C) External iliac nodes (D) Superficial inguinal nodes (E) Common iliac nodes

27. The answer is D. The superficial inguinal lymph nodes receive lymph from the scrotum, penis, buttocks, and lower part of the anal canal, and their efferent vessels enter primarily to the external iliac nodes and ultimately to the lumbar (aortic) nodes. The deep inguinal nodes receive lymph from the testis and upper parts of the vagina and anal canal, and their efferent vessels enter the external iliac nodes.

28. The Answer is D. The great saphenous nerve remains intact because it is not in the adductor canal. The adductor canal contains the femoral vessels, the saphenous nerve, and the nerve to the vastus medialis.

28. A 72-year-old woman complains of a cramp-like pain in her thigh and leg. She was diagnosed as having a severe intermit- tent claudication. Following surgery, an infection was found in the adductor canal, damaging the enclosed structures. Which of the following structures remains intact? (A) Femoral artery (B) Femoral vein (C) Saphenous nerve (D) Great saphenous vein (E) Nerve to the vastus medialis

28. A 16-year-old boy presents to the emergency department with rupture of the penile urethra. Extravasated urine from this injury can spread into which of the following structures? (A) Scrotum (B) Ischiorectal fossa (C) Pelvic cavity (D) Testis (E) Thigh

28. The answer is A. Extravasated urine from the penile urethra below the perineal membrane spreads into the superfi cial perineal space, scrotum, penis, and anterior abdominal wall. However, it does not spread into the testis, ischiorectal fossa, pelvic cavity, and thigh because Scarpa's fascia ends by fi rm attachment to the fascia lata of the thigh

28. A patient with cirrhosis is scheduled for liver transplant surgery. During the operation rounds, the transplant physician explains to his residents that one of the reasons a surgeon must pay close attention to the anatomic location of the liver is that this organ: (A) Receives blood only from the hepatic arteries (B) Manufactures red blood cells in an adult (C) Drains bile from the quadrate lobe into the right hepatic duct (D) Drains venous blood into the hepatic veins (E) Functions to concentrate and store bile

28. The answer is D. The liver receives blood from the hepatic artery and portal vein and drains its venous blood into the hepatic veins. The liver manufactures red blood cells in the fetus. The liver plays important roles in bile production and secretion. The quadrate lobe drains bile into the left hepatic duct, not the right hepatic duct, whereas the caudate lobe drains bile into the right and left hepatic ducts. The gallbladder functions to concentrate and store bile.

29. The Answer is C. A muscular spasm or hypertrophy of the extensor muscles of the leg may compress the anterior tibial artery, causing ischemia. The popliteal artery supplies mus- cles of the popliteal fossa. The deep femoral artery supplies deep muscles of the thigh. The posterior tibial and peroneal arteries supply muscles of the posterior and lateral compart- ments of the leg.

29. A basketball player was hit in the thigh by an opponent's knee. Which of the following arteries is likely to compress and cause ischemia because of the bruise and damage to the extensor muscles of the leg? (A) Popliteal (B) Deep femoral (C) Anterior tibial (D) Posterior tibial (E) Peroneal

29. A 41-year-old woman is brought to the emergency department by her family because of acute onset of right upper quadrant pain, nausea, and vomiting. For this case, it is important to remember that the bile duct: (A) Drains bile into the second part of the duodenum (B) Can be blocked by cancer in the body of the pancreas (C) Joins the main pancreatic duct, which carries hormones (D) Is formed by union of the right and left hepatic duct (E) Lies posterior to the portal vein in the right free edge of the lesser omentum

29. The answer is A. The bile duct is formed by union of the common hepatic and cystic ducts, lies lateral to the proper hepatic artery and anterior to the portal vein in the right free margin of the lesser omentum, traverses the head of the pancreas, and drains bile into the second part of the duodenum at the greater papilla. The endocrine part of the pancreas secretes the hormones insulin and glucagon, which are transported through the bloodstream. The main pancreatic duct carries pancreatic juice containing enzymes secreted from the exocrine part of the pancreas.

29. A 23-year-old woman visits her obstetrician for an annual checkup. During vaginal examination, which of the following structures may be palpated? (A) Apex of the urinary bladder (B) Fundus of the uterus (C) Terminal part of the round ligament of the uterus (D) Body of the clitoris (E) Uterine cervix

29. The answer is E. In addition to the uterine cervix, the uterus, uterine tubes, ovaries, and ureters can be palpated. The apex of the urinary bladder is the anterior end of the bladder; thus, it cannot be palpated. The fundus of the uterus is the anterosuperior part of the uterus. The terminal part of the round ligament of the uterus emerges from the superfi cial inguinal ring and becomes lost in the subcutaneous tissue of the labium majus.

3. The Answer is D. The posterior cruciate ligament is important because it prevents forward displacement of the femur on the tibia when the knee is flexed. The anterior cruciate liga- ment prevents backward displacement of the femur on the tibia.

3. A patient is unable to prevent anterior displacement of the femur on the tibia when the knee is flexed. Which of the following ligaments is most likely damaged? (A) Anterior cruciate (B) Fibular collateral (C) Patellar (D) Posterior cruciate (E) Tibial collateral

3. A 2-year-old boy presents with pain in his groin that has been increasing in nature over the past few weeks. He is found to have a degenerative malformation of the transversalis fascia during development. Which of the following structures on the anterior abdominal wall is likely defective? (A) Superficial inguinal ring (B) Deep inguinal ring (C) Inguinal ligament (D) Sac of a direct inguinal hernia (E) Anterior wall of the inguinal canal

3. The answer is B. The deep inguinal ring lies in the transversalis fascia, just lateral to the inferior epigastric vessels. The superficial inguinal ring is in the aponeurosis of the external oblique muscle. The inguinal ligament and the anterior wall of the inguinal canal are formed by the aponeurosis of the external oblique muscle. The sac of a direct inguinal hernia is formed by the peritoneum.

171. The internal capsule is composed of nerve fibers classified as: A. Projectional B. Short associational C. Commissural D. Arcuate E. Long associational

A. Projectional

30. The Answer is D. The gluteus maximus is inserted into the gluteal tuberosity of the femur and the iliotibial tract. All of the other muscles insert on the greater trochanter of the femur, and their functions are impaired.

30. An elderly woman fell at home and fractured the greater trochanter of her femur. Which of the following muscles would continue to function normally? (A) Piriformis (B) Obturator internus (C) Gluteus medius (D) Gluteus maximus (E) Gluteus minimus

30. A 53-year-old bank teller is admitted to a local hospital for surgical removal of a benign pelvic tumor confi ned within the broad ligament. There is a risk of injuring which of the following structures that lies in this ligament? (A) Ovary (B) Proximal part of the pelvic ureter (C) Terminal part of the round ligament of the uterus (D) Uterine tube (E) Suspensory ligament of the ovary

30. The answer is D. The uterine tubes lie in the broad ligament. The anterior surface of the ovary is attached to the posterior surface of the broad ligament of the uterus. The ureter descends retroperitoneally on the lateral pelvic wall but is crossed by the uterine artery in the base (in the inferomedial part) of the broad ligament. The terminal part of the round ligament of the uterus becomes lost in the subcutaneous tissue of the labium majus. The suspensory ligament of the ovary is a band of peritoneum that extends superiorly from the ovary to the pelvic wall.

30. A patient with diverticulosis of the colon presents for follow-up to his primary care physician with ongoing complaints of left lower quadrant pain and occasionally bloody stools. His physician begins workup with appropriating test by recalling that the sigmoid colon: (A) Is drained by systemic veins (B) Is a retroperitoneal organ (C) Receives parasympathetic fibers from the vagus nerve (D) Receives its blood from the superior mesenteric artery (E) Has teniae coli and epiploic appendages

30. The answer is E. The sigmoid colon has teniae coli and epiploic appendages. The sigmoid colon receives blood from the inferior mesenteric artery, drains its venous blood through the portal tributaries, has its own mesentery (sigmoid mesocolon, therefore, is not a retroperitoneal organ), and receives parasympathetic preganglionic fibers from the pelvic splanchnic nerve.

31. The Answer is C. The common peroneal nerve is vulnerable to injury as it passes behind the head of the fibula and then winds around the neck of the fibula and pierces the peroneus longus muscle, where it divides into the deep and superficial peroneal nerves. In addition, the deep and superficial peroneal nerves pass superficial to the neck of the fibula in the substance of the peroneus longus muscle and are less susceptible to injury than the common peroneal nerve. Other nerves are not closely associated with the head and neck of the fibula.

31. Radiographic examination reveals a fracture of the head and neck of the fibula. Which of the following nerves is damaged? (A) Sciatic (B) Tibial (C) Common peroneal (D) Deep peroneal (E) Superficial peroneal

31. A 72-year-old man comes to his physician for an annual checkup. Which of the following structures is most readily palpated during rectal examination? (A) Prostate gland (B) Epididymis (C) Ejaculatory duct (D) Ureter (E) Testis

31. The answer is A. The prostate gland may be palpated on rectal examination. The ejaculatory duct runs within the prostate gland and cannot be felt. In the male, the pelvic part ofthe ureter lies lateral to the ductus deferens and enters the posterosuperior angle of the bladder, where it is situated anterior to the upper end of the seminal vesicle, and thus, it cannot be palpated during rectal examination. However, in the female, the ureter can be palpated during vaginal examination because it runs near the uterine cervix and the lateral fornix of the vagina to enter the posterosuperior angle of the bladder. The testes are examined during a routine annual checkup but obviously not during a rectal examination.

31. A 19-year-old man with a ruptured appendix is sent to the emergency department for surgery. To cut off the blood supply to the appendix (if collateral circulation is discounted), a surgeon should ligate which of the following arteries? (A) Middle colic artery (B) Right colic artery (C) Ileocolic artery (D) Inferior mesenteric artery (E) Common iliac artery

31. The answer is C. The appendicular artery is a branch of the ileocolic artery. The other arteries do not supply the appendix. The middle colic and right colic arteries are branches of the superior mesenteric artery. The inferior mesenteric artery passes to the left behind the peritoneum and distributes to the descending and sigmoid colons and the upper portion of the rectum. The common iliac arteries are bifurcations from the aorta.

32. The Answer is C. The extensor hallucis longus is innervated by the deep peroneal nerve, whereas other muscles are innervated by the posterior tibial nerve.

32. After injury to the common peroneal nerve, which of the following muscles could be paralyzed? (A) Gastrocnemius (B) Popliteus (C) Extensor hallucis longus (D) Flexor digitorum longus (E) Tibialis posterior

32. A 48-year-old college football coach undergoes a radical prostatectomy for a malignant tumor in his prostate. Following surgery, he is incapable of achieving an erection. Which of the following nerves is most likely damaged during the surgery? (A) Sacral splanchnic nerve (B) Pelvic splanchnic nerve (C) Pudendal nerve (D) Dorsal nerve of the penis (E) Posterior scrotal nerve

32. The answer is B. Parasympathetic preganglionic fi bers in the pelvic splanchnic nerve are responsible for erection of the penis. Sympathetic preganglionic fi bers in the sacral splanchnic nerve are responsible for ejaculation. The pudendal nerve supplies the external anal sphincter and perineal muscles and supplies GSA fi bers to the perineal region. The dorsal nerve of the penis is a terminal branch of the pudendal nerve and supplies sensation of the penis. The posterior scrotal nerves are superfi cial branches of the perineal nerve and supply sensory fi bers to the scrotum.

32. Because of an inflammatory bowel disease (Crohn disease) and a small bowel obstruction leading to bowel ischemia, an elderly woman requires bypass of her ileum and jejunum and is scheduled for a gastrocolostomy. The surgeon will ligate all arteries that send branches to the stomach. Which of the following arteries may be spared? (A) Splenic artery (B) Gastroduodenal artery (C) Inferior pancreaticoduodenal artery (D) Left gastroepiploic artery (E) Proper hepatic artery

32. The answer is C. The inferior pancreaticoduodenal artery does not supply the stomach. All of the other arteries supply the stomach. Gastrocolostomy is used to establish a communication between the stomach and colon, bypassing the small intestine when the patient has Crohn disease (inflammation disease) and small bowel obstruction.

33. The Answer is B. The lateral (fibular) collateral ligament prevents adduction at the knee. Therefore, a torn lateral collateral ligament can be recognized by abnormal passive adduction of the extended leg. Abnormal passive abduction of the extended leg may occur when the medial (tibial) collateral ligament is torn. The anterior cruciate ligament prevents posterior displacement of the femur on the tibia; the posterior cruciate ligament prevents anterior displacement of the femur on the tibia. In addition, the posterior cruciate ligament is taut when the knee is fully flexed.

33. If the lateral (fibular) collateral ligament is torn by a fracture of the head and neck of the fibula, which of the following conditions may occur? (A) Abnormal passive abduction of the extended leg (B) Abnormal passive adduction of the extended leg (C) Anterior displacement of the femur on the tibia (D) Posterior displacement of the femur on the tibia (E) Maximal flexion of the leg

33. A 38-year-old woman with peptic ulcer disease of the stomach experiences severe abdominal pain. Which of the following nervous structures is most likely involved? (A) Greater splanchnic nerve (B) Ventral roots of the spinal nerve (C) Lower intercostal nerve (D) Vagus nerve (E) Gray ramus communicans

33. The answer is A. The greater splanchnic nerve carries pain fibers from the upper GI tract. Neither the ventral roots of the spinal nerves nor the gray rami communicantes contain sensory nerve fibers. The vagus nerve contains sensory fibers associated with reflexes, but it does not contain pain fibers. The lower intercostal nerves carry general somatic afferent (GSA) pain fibers from the diaphragm, abdominal wall, and peritoneum but not GVA pain fibers from the GI tract.

33. While performing a pelvic exenteration, the surgical oncologist notices a fractured or ruptured boundary of the pelvic inlet. Which of the following structures is most likely damaged? (A) Promontory of the sacrum (B) Anterior-inferior iliac spine (C) Inguinal ligament (D) Iliac crest (E) Arcuate pubic ligament

33. The answer is A. The pelvic inlet (pelvic brim) is bounded by the promontory and the anterior border of the ala of the sacrum, the arcuate line of the ilium, the pectineal line, the pubic crest, and the superior margin of the pubic symphysis.

34. A 32-year-old patient with multiple fractures of the pelvis has no cutaneous sensation in the urogenital triangle. The function of which of the following nerves is most likely to be spared? (A) Ilioinguinal nerve (B) Iliohypogastric nerve (C) Posterior cutaneous nerve of the thigh (D) Pudendal nerve (E) Genitofemoral nerve

34. The answer is B. The iliohypogastric nerve innervates the skin above the pubis. The skin of the urogenital triangle is innervated by the pudendal nerve, perineal branches of the posterior femoral cutaneous nerve, anterior scrotal or labial branches of the ilioinguinal nerve, and the genital branch of the genitofemoral nerve

34. A 3-year-old boy is diagnosed as having a persistent processus vaginalis in its middle portion. Which of the following conditions is most likely to be associated with this developmental anomaly? (A) Direct inguinal hernia (B) Gubernaculum testis (C) Hematocele (D) Hydrocele (E) Cryptorchidism

34. The answer is D. If a middle portion of the processus vaginalis persists, it forms a congenital hydrocele. If the entire processus vaginalis persists, it develops a congenital indirect inguinal hernia. Gubernaculum testis is the fetal ligament that connects the bottom of the fetal testis to the developing scrotum. Hematocele is an effusion of blood into the cavity of the tunica vaginalis. Cryptorchidism is failure of the testis to descend from the abdomen to the scrotum.

34. The Answer is C. The anterior tibial artery, which arises from the popliteal artery, enters the anterior compartment by passing through the gap between the fibula and tibia at the upper end of the interosseous membrane. The other arteries would not be affected because they are not closely associated with the head and neck of the fibula.

34. Which of the following arteries could be damaged by a fracture of the head and neck of the fibula? (A) Popliteal (B) Posterior tibial (C) Anterior tibial (D) Peroneal (E) Lateral inferior genicular

35. A 22-year-old victim of an automobile accident has received destructive damage to structures that form the boundary of the perineum. Which of the following structures is spared? (A) Pubic arcuate ligament (B) Tip of the coccyx (C) Ischial tuberosities (D) Sacrospinous ligament (E) Sacrotuberous ligament

35. The answer is D. The sacrospinous ligament forms a boundary of the lesser sciatic foramen. The pubic arcuate ligament, tip of the coccyx, ischial tuberosities, and sacrotuberous ligament all form part of the boundary of the perineum.

35. Examination of a 54-year-old man reveals an isolated tumor located at the porta hepatis. This tumor most likely compresses which of the following structures? (A) Cystic duct (B) Hepatic veins (C) Common hepatic artery (D) Left gastric artery (E) Branches of the portal vein

35. The answer is E. The porta hepatis is the transverse fissure (doorway) in the liver and contains the hepatic ducts, hepatic arteries, and branches of the portal vein. The other structures are not found in the porta hepatis.

35. The Answer is D. Anterior tibial compartment syndrome is characterized by ischemic necrosis of the muscles of the anterior tibial compartment of the leg resulting from dam- age to the anterior tibial artery. The gastrocnemius receives blood from sural branches of the popliteal artery. Loss of plantar flexion is due to necrosis of the posterior muscles of the leg, which are supplied by the posterior tibial and peroneal arteries. Trendelenburg's sign is caused by weakness or paralysis of the gluteus medius and minimus muscles. Flat foot results from the collapse of the medial longitudinal arch of the foot.

35. Which of the following conditions would occur from a fracture of the head and neck of the fibula? (A) Ischemia in the gastrocnemius (B) Loss of plantar flexion (C) Trendelenburg's sign (D) Anterior tibial compartment syndrome (E) Flat foot

36. The Answer is C. The tibialis posterior can plantar flex and invert the foot. The extensor digitorum longus can dorsiflex and evert the foot, the tibialis anterior can dorsiflex and invert the foot, and the peroneus longus and brevis can plantar flex and evert the foot.

36. A construction worker is hit on the leg with a concrete block and is subsequently unable to plantar flex and invert his foot. Which of the following muscles is most likely damaged? (A) Extensor digitorum longus (B) Tibialis anterior (C) Tibialis posterior (D) Peroneus longus (E) Peroneus brevis

36. A 32-year-old man undergoes vasectomy as a means of permanent birth control. A physician performing the vasectomy by making an incision on each side of the scrotum should remember which of the following statements most applicable to the scrotum? (A) It is innervated by the ilioinguinal and genitofemoral nerves. (B) It receives blood primarily from the testicular artery. (C) Its venous blood drains primarily into the renal vein on the left. (D) Its lymphatic drainage is primarily into upper lumbar nodes. (E) Its dartos tunic is continuous with the perineal membrane.

36. The answer is A. The scrotum is innervated by branches of the ilioinguinal, genitofemoral, pudendal, and posterior femoral cutaneous nerves. The scrotum receives blood from the posterior scrotal branches of the internal pudendal arteries and the anterior scrotal branches of the external pudendal arteries, but it does not receive blood from the testicular artery. Similarly, the scrotum is drained by the posterior scrotal veins into the internal pudendal vein. The lymph vessels from the scrotum drain into the superfi cial inguinal nodes, whereas the lymph vessels from the testis drain into the upper lumbar nodes. The dartos tunic is continuous with the membranous layer of the superfi cial perineal fascia (Colles's fascia).

36. A patient is rushed to the operating room for an emergent cholecystectomy (resection of a gallbladder) because of cholecystitis. While locating landmarks before surgical resection of an infected gallbladder, the surgeon recalls a portal-caval anastomosis. Which of the following pairs of veins form a portal-caval anastomosis? (A) Hepatic veins and IVC (B) Superior and middle rectal vein (C) Left and right gastric vein (D) Inferior and superficial epigastric veins (E) Suprarenal and renal veins

36. The answer is B. Portal-caval anastomoses occur between the left gastric vein and esophageal vein of the azygos, the superior rectal and middle or inferior rectal veins, paraumbilical and superficial epigastric veins, and retrocolic veins and twigs of the renal vein. The hepatic veins and the IVC are systemic or caval veins. The left and right gastric veins belong to the portal venous system. The inferior and superficial epigastric veins and the suprarenal and renal veins are systemic veins.

37. A 37-year-old woman complains of a bearing-down sensation in her womb and an increased frequency of and burning sensation on urination. On examination by her gynecologist, she is diagnosed with a uterine prolapse. Which of the following structures provides the primary support for the cervix of the uterus? (A) External anal sphincter (B) Broad ligament of the uterus (C) Cardinal (transverse cervical) ligament (D) Round ligament of the uterus (E) Suspensory ligament of the ovary

37. The answer is C. The cardinal (transverse cervical) ligament provides the major ligamentous support for the uterus. The sphincter ani externus does not support the uterus. The broad and round ligaments of the uterus provide minor supports for the uterus. The suspensory ligament of the ovary does not support the uterus.

37. Mrs. Jones is undergoing a routine colonoscopy for colon cancer prevention. The gastroenterologist finds a Meckel diverticulum. Which of the following statements is true about the diverticulum? (A) It is found 2 ft distal to the ileocecal junction (B) It is located on the mesenteric side of the ileum (C) It occurs in approximately 20% of the population (D) It is a persistent remnant of the embryonic yolk stalk (E) It may contain renal and suprarenal tissues

37. The answer is D. The Meckel diverticulum is a persistent remnant of the yolk stalk (vitelline duct) and located 2 ft proximal to the ileocecal junction on the antimesenteric border of the ileum. It is approximately 2 in. long, occurs in approximately 2% of the population, and contains two types of mucosal (gastric and pancreatic) tissues in its wall.

37. The Answer is D. The adductor magnus is innervated by both the obturator and sciatic (tibial portion) nerves. Hence, a lesion here could cause paralysis. The rectus femoris and sartorius are innervated by the femoral nerve. The biceps femoris long head is innervated by the tibial portion of the sciatic nerve, whereas the short head is innervated by the com- mon peroneal portion of the sciatic nerve. The pectineus is innervated by both the femoral and obturator nerves.

37. The obturator nerve and the sciatic (tibial portion) nerve of a 15-year-old boy are transected as a result of a motorcycle accident. This injury would result in complete paralysis of which of the following muscles? (A) Rectus femoris (B) Biceps femoris, short head (C) Pectineus (D) Adductor magnus (E) Sartorius

38. The Answer is C. The rectus femoris flexes the thigh and extends the leg. The sartorius can flex both the hip and knee joints. The gracilis adducts and flexes the thigh and flexes the leg, the vastus medialis extends the knee joint, and the semimembranosus extends the hip joint and flexes the knee joint.

38. A 24-year-old woman presents to her physician with weakness in flexing the hip joint and extending the knee joint. Which muscle is most likely involved in this scenario? (A) Sartorius (B) Gracilis (C) Rectus femoris (D) Vastus medialis (E) Semimembranosus

38. A woman is delivering a breech baby. The obstetrician decides that it is best to perform a mediolateral episiotomy. Which of the following structures should the obstetrician avoid incising? (A) Vaginal wall (B) Superfi cial transverse perineal muscle (C) Bulbospongiosus (D) Levator ani (E) Perineal membrane

38. The answer is D. An obstetrician should avoid incising the levator ani and the external anal sphincter. The levator ani is the major part of the pelvic diaphragm, which forms the pelvic fl oor and supports all of the pelvic organs. None of the other choices applies here.

38. A 54-year-old man comes to a hospital with abdominal pain, jaundice, loss of appetite, and weight loss. On examination of his radiograms and CT scans, a physician finds a slowly growing tumor in the uncinate process of the pancreas. Which of the following structures is most likely compressed by this tumor? (A) Main pancreatic duct (B) Splenic artery (C) Portal vein (D) Superior mesenteric artery (E) Superior pancreaticoduodenal artery

38. The answer is D. The uncinate process of the pancreas is a projection of the lower part of its head to the left behind the superior mesenteric vessels. The superior pancreaticoduodenal artery runs between the duodenum and the head of the pancreas. The main pancreatic duct runs transversely through the entire pancreas superior to the uncinate process. The splenic artery runs along the superior border of the pancreas. The portal vein runs behind the neck of the pancreas.

39. The Answer is C. The adductor longus is innervated by only the obturator nerve. Thus, injury here could completely paralyze the adductor longus. The pectineus is innervated by both the obturator and femoral nerves. The adductor magnus is innervated by both the obturator nerve and tibial part of the sciatic nerve. The biceps femoris is innervated by the tibial portion (long head) and common peroneal portion (short head) of the sciatic nerve. The semimembranosus is innervated by the tibial portion of the sciatic nerve.

39. A 17-year-old boy was stabbed during a gang fight resulting in transection of the obturator nerve. Which of the following muscles is completely paralyzed? (A) Pectineus (B) Adductor magnus (C) Adductor longus (D) Biceps femoris (E) Semimembranosus

39. During pelvic surgery, a surgeon notices severe bleeding from the artery that remains within the true pelvis. Which of the following arteries is most likely to be injured? (A) Iliolumbar artery (B) Obturator artery (C) Uterine artery (D) Internal pudendal artery (E) Inferior gluteal artery

39. The answer is C. Of all the arteries listed, the uterine artery remains within the pelvic cavity.

39. A 6-year-old boy comes to his pediatrician with a lump in the groin near the thigh and pain in the groin. On examination, the physician makes a diagnosis of a direct inguinal hernia because the herniated tissue: (A) Enters the deep inguinal ring (B) Lies lateral to the inferior epigastric artery (C) Is covered by spermatic fasciae (D) Descends into the scrotum (E) Develops after birth

39. The answer is E. A direct hernia is acquired (develops after birth), whereas an indirect inguinal hernia is congenital. The direct hernia does not enter the deep inguinal ring but occurs through the posterior wall of the inguinal canal, lies medial to the inferior epigastric artery, is covered only by peritoneum, and does not descend into the scrotum.

4. The answer is D. The femoral nerve innervates the quadratus femoris, sartorius, and vastus muscles. Therefore, damage to this nerve results in paralysis of these muscles. The second and third lumbar nerves innervate the psoas major muscle, the sural nerve innervates the skin on the lateral side of the foot, the iliohypogastric nerve and superior clunial nerves supply the skin over the greater trochanter, and the superior gluteal nerve innervates the tensor fasciae latae.

4. A 41-year-old man was involved in a fight and felt weakness in extending the knee joint. On examination, he was diagnosed with a lesion of the femoral nerve. Which of the following symptoms would be a result of this nerve damage? (A) Paralysis of the psoas major muscle (B) Loss of skin sensation on the lateral side of the foot (C) Loss of skin sensation over the greater trochanter (D) Paralysis of the vastus lateralis muscle (E) Paralysis of the tensor fasciae latae

4. A 29-year-old man comes to a local hospital with duodenal peptic ulcer and complains of cramping epigastric pain. Which of the following structures harbors the cell bodies of abdominal pain fibers? (A) Lateral horn of the spinal cord (B) Anterior horn of the spinal cord (C) Dorsal root ganglion (D) Sympathetic chain ganglion (E) Celiac ganglion

4. The answer is C. Cell bodies of the abdominal pain fibers are located in the dorsal root ganglion. The lateral horn of the spinal cord contains cell bodies of sympathetic preganglionic nerve fibers; the anterior horn contains cell bodies of general somatic efferent (GSE) fibers. The sympathetic chain ganglion contains cell bodies of sympathetic postganglionic fibers, which supply blood vessels, sweat glands, and hair follicles. The celiac ganglion contains cell bodies of sympathetic postganglionic fibers, which supply the visceral organs such as stomach and intestine.

4. A 22-year-old woman receives a deep cut in the inguinal canal 1 in. lateral to the pubic tubercle. Which of the following ligaments is lacerated within the inguinal canal? (A) Suspensory ligament of the ovary (B) Ovarian ligament (C) Mesosalpinx (D) Round ligament of the uterus (E) Rectouterine ligament

4. The answer is D. The round ligament of the uterus is found in the inguinal canal along its course. The other ligaments do not pass through the inguinal canal.

40. The Answer is B. The keystone for the lateral longitudinal arch is the cuboid bone, whereas the keystone for the medial longitudinal arch is the head of the talus. The calcaneus, navic- ular, and medial cuneiform bones form a part of the medial longitudinal arch, but they are not keystones. The calcaneus also forms a part of the lateral longitudinal arch.

40. A 32-year-old carpenter fell from the roof. The lateral longitudinal arch of his foot was flattened from fracture and displace- ment of the keystone for the arch. Which of the following bones is damaged? (A) Calcaneus (B) Cuboid bone (C) Head of the talus (D) Medial cuneiform (E) Navicular bone

40. A neurosurgeon performs surgical resection of a rare meningeal tumor in the sacral region. He tries to avoid an injury of the nerve that arises from the lumbosacral plexus and remains within the abdominal or pelvic cavity. To which of the following nerves should he pay particular attention? (A) Ilioinguinal nerve (B) Genitofemoral nerve (C) Lumbosacral trunk (D) Femoral nerve (E) Lateral femoral cutaneous nerve

40. The answer is C. The lumbosacral trunk is formed by part of the ventral ramus of the fourth lumbar nerve and the ventral ramus of the fi fth lumbar nerve. This trunk contributes to the formation of the sacral plexus by joining the ventral ramus of the fi rst sacral nerve in the pelvic cavity and does not leave the pelvic cavity. All other nerves leave the abdominal and pelvic cavities

40. A 21-year-old man developed a hernia after lifting heavy boxes while moving into his new house. During the repair of his resulting hernia, the urologist recalls that the genitofemoral nerve: (A) Runs in front of the quadratus lumborum (B) Is a branch of the femoral nerve (C) Supplies the testis (D) Passes through the deep inguinal ring (E) Gives rise to an anterior scrotal branch

40. The answer is D. The genitofemoral nerve descends on the anterior surface of the psoas muscle and gives rise to a genital branch, which enters the inguinal canal through the deep inguinal ring to supply the cremaster muscle, and a femoral branch, which supplies the skin of the femoral triangle. The genitofemoral nerve is not a branch of the femoral nerve but arises from the lumbar plexus and does not supply the testis. It is the ilioinguinal nerve that gives rise to an anterior scrotal branch.

41. An oncologist is reviewing a CT scan of a 74-year-old man with newly diagnosed hepatocellular carcinoma. He locates the affected quadrate lobe of the liver that: (A) Lies between the IVC and ligamentum venosum (B) Receives blood from the right hepatic artery (C) Drains bile into the left hepatic duct (D) Is a medial superior segment (E) Is functionally a part of the right lobe

41. The answer is C. The quadrate lobe of the liver drains bile into the left hepatic duct and receives blood from the left hepatic artery. It lies between the gallbladder fossa and the ligamentum teres hepatic, is a medial inferior segment, and is a part of the left lobe.

41. After repair of a ruptured diverticulum, a 31-year-old patient begins to spike with fever and complains of abdominal pain. An infection in the deep perineal space would most likely damage which of the following structures? (A) Ischiocavernosus muscles (B) Superfi cial transverse perineal muscles (C) Levator ani (D) Sphincter urethrae (E) Bulbospongiosus

41. The answer is D. The sphincter urethrae is found in the deep perineal space, whereas the other structures are located in the superfi cial perineal space.

41. The Answer is A. The "unhappy triad" of the knee joint is characterized by tear of the medial meniscus, rupture of the tibial collateral ligament, and rupture of the anterior cruciate ligament. This injury may occur when a cleated shoe, as worn by football players, is planted firmly in the turf and the knee is struck from the lateral side. Tenderness along the medial collateral ligament and over the medial meniscus and swelling on the front of the joint are due to excessive production of synovial fluid, which fills the joint cavity and the suprapatellar bursa.

41. While playing football, a 19-year-old college student receives a twisting injury to his knee when being tackled from the lateral side. Which of the following conditions most likely has occurred? (A) Tear of the medial meniscus (B) Ruptured fibular collateral ligament (C) Tenderness on pressure along the fibular collateral ligament (D) Injury of the posterior cruciate ligament (E) Swelling on the back of the knee joint

42. The Answer is D. The sartorius can flex and rotate the thigh laterally, and flex and rotate the leg medially. The rectus femoris flexes the thigh and extends the leg. The semimembranosus extends the thigh and flexes and rotates the leg medially. The biceps femoris extends the thigh and flexes and rotates the leg laterally. The adductor longus adducts and flexes the thigh.

42. A patient has weakness when flexing both her thigh and leg. Which of the following muscles is most likely injured? (A) Rectus femoris (B) Semitendinosus (C) Biceps femoris (D) Sartorius (E) Adductor longus

42. A 58-year-old man is presented with edema of the lower limb and enlarged superficial veins of the abdominal wall. Examination of radiographs and angiograms reveals obstruction of the IVC just proximal to the entrance of the renal vein. This venous blockage may result in dilation of which of the following veins? (A) Left suprarenal vein (B) Right inferior phrenic vein (C) Right hepatic vein (D) Left gastric vein (E) Portal vein

42. The answer is A. The veins distal to obstruction are dilated, but the veins proximal to obstruction are not dilated but have low blood pressure. The suprarenal vein drains into the left renal vein and thus is dilated because of high pressure. The right phrenic and right hepatic veins drain into the IVC above the obstruction. The left gastric vein joins the portal vein, which enters the liver.

42. A radiologist interprets a lymphangiogram for a 29-year-old patient with metastatic carcinoma. Upper lumbar nodes most likely receive lymph from which of the following structures? (A) Lower part of the anal canal (B) Labium majus (C) Clitoris (D) Testis (E) Scrotum

42. The answer is D. Lymphatic vessels from the testis and epididymis ascend along the testicular vessels in the spermatic cord through the inguinal canal and continue upward in the abdomen to drain into the upper lumbar nodes. The lymph from the other structures drains into the superfi cial inguinal lymph nodes.

43. The Answer is C. The tibialis anterior muscle can dorsiflex the foot, whereas all other muscles are able to plantar flex the foot.

43. A 35-year-old man has difficulty in dorsi- flexing the foot. Which of the following muscles is most likely damaged? (A) Tibialis posterior (B) Flexor digitorum longus (C) Tibialis anterior (D) Peroneus longus (E) Peroneus brevis

190. The major venous system of the heart, the coronary sinus, empties into the: A. Right atrium B. Left atrium C. Right ventricle D. Superior vena cava E. Inferior vena cava

A. Right atrium

43. A physical fitness trainer for a young Hollywood movie star explains the reasons for 100 stomach crunches a day. The young star, a medical student before 'hitting it big,' reaffirms to his trainer that the lateral margin of the rectus abdominis, the muscle responsible for a washboard stomach, defines which of the following structures? (A) Linea alba (B) Linea semilunaris (C) Linea semicircularis (D) Transversalis fascia (E) Falx inguinalis

43. The answer is B. The linea semilunaris is a curved line along the lateral border of the rectus abdominis. The linea alba is a tendinous median raphe between the two rectus abdominis muscles. The linea semicircularis is an arcuate line of the rectus sheath, which is the lower limit of the posterior layer of the rectus sheath. The falx inguinalis (conjoint tendon) is formed by aponeuroses of the internal oblique and transverse abdominal muscles (otherwise known as the transversalis fascia).

43. A 49-year-old woman has a large mass on the pelvic brim. Which of the following structures is most likely compressed by this mass when crossing the pelvic brim? (A) Deep dorsal vein of the penis (B) Uterine tube (C) Ovarian ligament (D) Uterine artery (E) Lumbosacral trunk

43. The answer is E. All of the listed structures do not cross the pelvic brim except the lumbosacral trunk, which arises from L4 and L5, enters the true pelvis by crossing the pelvic brim, and contributes to the formation of the sacral plexus. The deep dorsal vein of the penis enters the pelvic cavity by passing under the symphysis pubis between the arcuate and transverse perineal ligaments.

44. The Answer is A. The tibialis posterior inverts the foot. The peroneus longus, brevis, and tertius and extensor digitorum longus can evert the foot.

44. An injury to the leg of a golfer results in loss of the ability to invert the foot. Which of the following muscles is most likely paralyzed? (A) Tibialis posterior (B) Peroneus longus (C) Peroneus brevis (D) Peroneus tertius (E) Extensor digitorum longus

44. A 26-year-old man comes to a hospital with fever, nausea, pain, and itching in the perineal region. On examination by a urologist, he is diagnosed as having infected bulbourethral (Cowper's) glands. Which of the following structures is/are affected by this infection? (A) Superfi cial perineal space (B) Sphincter urethrae (C) Production of sperm (D) Testis (E) Seminal vesicles

44. The answer is B. The bulbourethral glands lie on either side of the membranous urethra, embedded in the sphincter urethrae. Their ducts open into the bulbous part of the penile urethra. Semen—a thick, yellowish-white, viscous, spermatozoa-containing fl uid—is a mixture of the secretions of the testes, seminal vesicles, prostate, and bulbourethral glands. Sperm, or spermatozoa, are produced in the seminiferous tubules of the testis and mature in the head of the epididymis. The seminal vesicles are lobulated glandular structures, produce the alkaline constituent of the seminal fl uid that contains fructose and choline, and lie inferior and lateral to the ampullae of the ductus deferens against the fundus (base) of the bladder.

44. During surgical treatment of portal hypertension in a 59-year-old man with liver cirrhosis, a surgeon inadvertently lacerates the dilated paraumbilical veins. The veins must be repaired to allow collateral flow. Which of the following ligaments is most likely severed? (A) Lienorenal ligament (B) Lienogastric ligament (C) Gastrophrenic ligament (D) Ligamentum teres hepatis (E) Ligamentum venosum

44. The answer is D. The paraumbilical veins and the ligamentum teres hepatis are contained in the free margin of the falciform ligament. The lienorenal ligament contains the splenic vessels and a small portion of the tail of the pancreas. The lienogastric ligament contains the left gastroepiploic and short gastric vessels. The gastrophrenic ligament contains no named structures. The hepatoduodenal ligament, a part of the lesser omentum, contains the bile duct, proper hepatic artery, and portal vein in its free margin.

45. The Answer is A. The peroneal artery is a branch of the posterior tibial artery. The dorsalis pedis artery begins anterior to the ankle as the continuation of the anterior tibial artery. The superior medial genicular artery is a branch of the popliteal artery, and the descending genicular artery arises from the femoral artery.

45. An orthopedic surgeon ligates the posterior tibial artery at its origin. Which of the following arteries has no blood flow immediately after the ligation? (A) Peroneal (B) Dorsalis pedis (C) Superior medial genicular (D) Anterior tibial (E) Descending genicular

45. A 21-year-old man is involved in a highspeed motor vehicle accident. As a result, he has extensive damage to his sphincter urethrae. Which of the following best describes the injured sphincter urethrae? (A) Smooth muscle (B) Innervated by the perineal nerve (C) Lying between the perineal membrane and Colles's fascia (D) Enclosed in the pelvic fascia (E) Part of the pelvic diaphragm

45. The answer is B. The sphincter urethrae is striated muscle that lies in the deep perineal space and forms a part of the urogenital diaphragm but not the pelvic diaphragm. It is not enclosed in the pelvic fascia. It is innervated by a deep (muscular) branch of the perineal nerve.

45. A 43-year-old woman is admitted to the hospital because of deep abdominal pain in her epigastric region. On examination, it is observed that a retroperitoneal infection erodes an artery that runs along the superior border of the pancreas. Which of the following arteries is likely injured? (A) Right gastric artery (B) Left gastroepiploic artery (C) Splenic artery (D) Gastroduodenal artery (E) Dorsal pancreatic artery

45. The answer is C. The splenic artery arises from the celiac trunk, runs along the superior border of the pancreas, and enters the spleen through the lienorenal ligament and the hilus of the spleen. The right gastric artery runs along the lesser curvature of the stomach, and the left gastroepiploic artery runs along the greater curvature of the stomach. The gastroduodenal artery runs behind the first part of the duodenum. The dorsal pancreatic artery descends behind the neck of the pancreas and divides into right and left branches to supply the pancreas.

46. The Answer is D. The descending genicular artery gives off the articular branch, which enters the anastomosis around the knee joint, and the saphenous branch, which is not involved in the anastomosis but supplies the superficial tissue and skin on the medial side of the knee. Other arteries are involved in the anastomosis around the knee joint.

46. Before knee surgery, a surgeon ligates arteries participating in the anastomosis around the knee joint. Which of the following arteries is most likely spared? (A) Lateral superior genicular (B) Medial inferior genicular (C) Descending branch of the lateral femoral circumflex (D) Saphenous branch of the descending genicular (E) Anterior tibial recurrent

46. A 6 month-old male infant is admitted to the children's hospital because he has notestis in his scrotum. During physical examination, the pediatrician palpated the testis in the inguinal canal. What is the diagnosis of this condition? (A) Male pseudohermaphroditism (B) Hypospadias (C) Epispadias (D) Cryptorchid testis (E) Chordee

46. The answer is D. Cryptorchid testis is called an undescended testis, which is located in the inguinal region. Male pseudohermaphroditism is a condition in which the affected individual is a genetic and gonadal male with genital anomalies. Hypospadias occurs when the spongy urethra opens on the underside of the penis, frequently associated with the chordee, which is a ventral curvature of the penis. Epispadias occurs when the urethra opens on the dorsal surface of the penis.

46. A 19-year-old young woman with a long history of irritable bowel syndrome presents for the possibility of surgical resection of the gastrointestinal (GI) tract where the vagal parasympathetic innervation terminates. Which of the following sites is most appropriate for surgical resection? (A) Duodenojejunal junction (B) Ileocecal junction (C) Right colic flexure (D) Left colic flexure (E) Anorectal junction

46. The answer is D. The vagus nerve supplies parasympathetic nerve fibers to the GI tract and terminates approximately at the left colic flexure (junction of the transverse colon and the descending colon). The duodenojejunal junction, ileocecal junction, and right colic flexure are supplied by the vagus nerve. The descending colon, sigmoid colon, rectum, anal canal, and anorectal junction are supplied by the pelvic splanchnic nerve for parasympathetic innervation.

47. The Answer is B. The peroneus brevis muscle is innervated by the superficial peroneal nerve. The peroneus tertius and tibialis anterior muscles are innervated by the deep peroneal nerve. The flexor hallucis longus and tibialis posterior muscles are innervated by the tibial nerve.

47. A 25-year-old gladiator sustains a penetrating injury that severs the superficial peroneal nerve. This will most likely cause paralysis of which of the following muscles? (A) Peroneus tertius (B) Peroneus brevis (C) Flexor hallucis longus (D) Tibialis anterior (E) Tibialis posterior

47. A 58-year-old man is admitted to the hospital with severe abdominal pain, nausea, and vomiting resulting in dehydration. Emergency CT scan reveals a tumor located between the celiac trunk and the superior mesenteric artery. Which of the following structures is likely compressed by this tumor? (A) Fundus of the stomach (B) Neck of the pancreas (C) Transverse colon (D) Hepatopancreatic ampulla (E) Duodenojejunal junction

47. The answer is B. The pyloric canal and the neck of the pancreas are situated anterior to the abdominal aorta between the origin of the celiac trunk and the superior mesenteric artery. The transverse colon passes anterior to the superior mesenteric artery and the third part of the duodenum. The other structures are not located in front of the aorta.

47. An obstetrician is about to perform a pudendal block so a woman can experience less pain when she delivers her child. He recalls what he learned in medical school about this nerve. Which of the following statements is correct? (A) It passes superfi cial to the sacrotuberous ligament. (B) It innervates the testis and epididymis in a male. (C) It provides motor fi bers to the coccygeus. (D) It can be blocked by injecting an anesthetic near the inferior margin of the ischial spine. (E) It arises from the lumbar plexus

47. The answer is D. The pudendal nerve, which arises from the sacral plexus, provides sensory innervation to the labium majus (or scrotum in a male). It leaves the pelvis through the greater sciatic foramen and enters the perineum through the lesser sciatic foramen near the inferior margin of the ischial spine. Therefore, it can be blocked by injection of an anesthetic near the inferior margin of the ischial spine

48. The Answer is C. The popliteal vein drains blood into the femoral vein; thus, blood flow in the femoral vein is reduced. The great saphenous vein drains into the upper part of the femoral vein. Other veins empty into the popliteal vein.

48. A patient presents with a thrombosis in the popliteal vein. This thrombosis most likely causes reduction of blood flow in which of the following veins? (A) Greater saphenous (B) Lesser saphenous (C) Femoral (D) Posterior tibial (E) Anterior tibial

48. A trauma surgeon in the emergency department at a local center examines a 14-year-old boy with extensive pelvic injuries after a hit and run accident. The surgeon inspects the ischiorectal fossa because it: (A) Accumulates urine leaking from rupture of the bulb of the penis (B) Contains the inferior rectal vessels (C) Has a pudendal canal along its medial wall (D) Is bounded anteriorly by the sacrotuberous ligament (E) Contains a perineal branch of the fi fth lumbar nerve

48. The answer is B. The ischiorectal fossa is bounded posteriorly by the gluteus maximus and the sacrotuberous ligament. It contains fat, the inferior rectal nerve and vessels, and perineal branches of the posterior femoral cutaneous nerve. The pudendal canal runs along its lateral wall. Urine leaking from a ruptured bulb of the penis does not spread into the ischiorectal fossa because Scarpa's fascia ends by fi rm attachment to the fascia lata of the thigh.

48. An emergent hernia repair is scheduled. As the attending physician is driving to the hospital, the medical student assisting on the case quickly reviews his anatomy atlas and is trying to commit to memory that the internal oblique abdominis muscle contributes to the formation of which of the following structures? (A) Inguinal ligament (B) Deep inguinal ring (C) Falx inguinalis (conjoint tendon) (D) Internal spermatic fascia (E) Reflected inguinal ligament

48. The answer is C. The falx inguinalis (conjoint tendon) is formed by the aponeuroses of the internal oblique and transverse muscles of the abdomen. The inguinal ligament is formed by aponeurosis of the external oblique abdominal muscle, and the reflected inguinal ligament is formed by certain fibers of the inguinal ligament reflected from the pubic tubercle upward toward the linea alba. The deep inguinal ring lies in the transversalis fascia, and the internal spermatic fascia is formed by the transversalis fascia.

49. The Answer is D. The lateral (fibular) collateral ligament extends between the lateral femoral epicondyle and the head of the fibula and is not attached to the lateral meniscus. All other ligaments lie outside the synovial cavity but within the joint capsule.

49. A 21-year-old tennis player comes to an emergency room and complains of pain in the knee joint. On examination, he has an infection inside the knee joint capsule but outside the synovial cavity. Which of the following structures is preserved from this infection? (A) Anterior cruciate ligament (B) Posterior cruciate ligament (C) Lateral meniscus (D) Lateral collateral ligament (E) Medial meniscus

275. The sinoatrial node is located in the: A. Right atrium B. Left atrium C. Interventricular septum D. Left ventricle E. None of the above

A. Right atrium

49. An elderly man with prostatitis is seen at an internal medicine clinic. The seminal colliculus of his prostate gland is infected, and its fi neopenings are closed. Which of the following structures is/are most likely to be disturbed? (A) Ducts of the prostate gland (B) Prostatic utricle (C) Ducts of the bulbourethral glands (D) Ejaculatory ducts (E) Duct of the seminal vesicles

49. The answer is D. The ejaculatory ducts, which open onto the seminal colliculus, may be injured. The prostate ducts open into the urethral sinus, the bulbourethral ducts open into the bulbous part of the penile urethra, and the ducts of the seminal vesicle join the ampulla of the ductus deferens to form the ejaculatory duct. The prostatic utricle is a minute pouch on the summit of the seminal colliculus

49. A 9-year-old girl has crashed into her neighbor's brick fence while riding her bike and is brought to the emergency department with a great deal of abdominal pain. Her radiogram and angiogram show laceration of the superior mesenteric artery immediately distal to the origin of the middle colic artery. If collateral circulation is discounted, which of the following organs may become ischemic? (A) Descending colon (B) Duodenum (C) Pancreas (D) Ascending colon (E) Transverse colon

49. The answer is D. The right colic and ileocolic arteries arise from the superior mesenteric artery distal to the origin of the middle colic artery. The right colic artery may arise from the ileocolic artery and supplies the ascending colon. The duodenum and pancreas receive blood from the inferior pancreaticoduodenal artery and superior pancreaticoduodenal artery. The pancreas is also supplied by the splenic artery of the celiac trunk. The transverse colon receives blood from the middle colic artery. The descending colon is supplied by the left colic artery, which is a branch of the inferior mesenteric artery.

5. The Answer is B. The deep peroneal and tibial nerves innervate the chief evertors of the foot, which are the tibialis anterior, tibialis posterior, triceps surae, and extensor hallucis longus muscles. The tibialis anterior and extensor hallucis longus muscles are innervated by the deep peroneal nerve, and the tibialis posterior and triceps surae are innervated by the tibial nerve.

5. A 47-year-old woman is unable to invert her foot after she stumbled on her driveway. Which of the following nerves are most likely injured? (A) Superficial and deep peroneal (B) Deep peroneal and tibial (C) Superficial peroneal and tibial (D) Medial and lateral plantar (E) Obturator and tibial

5. A 29-year-old carpenter sustains severe injuries of the pelvic splanchnic nerve by a deep puncture wound, which has become contaminated. The injured parasympathetic preganglionic fi bers in the splanchnic nerve are most likely to synapse in which of the following ganglia? (A) Ganglia in or near the viscera or pelvic plexus (B) Sympathetic chain ganglia (C) Collateral ganglia (D) Dorsal root ganglia (E) Ganglion impar

5. The answer is A. The pelvic splanchnic nerves carry preganglionic parasympathetic general visceral efferent fi bers that synapse in the ganglia of the inferior hypogastric plexus and in terminal ganglia in the muscular walls of the pelvic organs. The sympathetic preganglionic fi bers synapse in the sympathetic chain (paravertebral) ganglia or in the collateral (prevertebral) ganglia. The dorsal root ganglia contain cell bodies of general somatic afferent (GSA) and general visceral afferent (GVA) fi bers and have no synapsis. The two sympathetic trunks unite and terminate in the ganglion impar (coccygeal ganglion), which is the most inferior, unpaired ganglion located in front of the coccyx.

5. A 42-year-old obese woman with seven children is brought to a local hospital by her daughter. Physical examination and her radiograph reveal that large gallstones have ulcerated through the posterior wall of the fundus of the gallbladder into the intestine. Which of the following parts of the intestine is most likely to initially contain gallstones? (A) Cecum (B) Ascending colon (C) Transverse colon (D) Descending colon (E) Sigmoid colon

5. The answer is C. The fundus of the gallbladder is in contact with the transverse colon, and thus, gallstones erode through the posterior wall of the gallbladder and enter the transverse colon. They are passed naturally to the rectum through the descending colon and sigmoid colon. Gallstones lodged in the body of the gallbladder may ulcerate through the posterior wall of the body of the gallbladder into the duodenum (because the gallbladder body is in contact with the duodenum) and may be held up at the ileocecal junction, producing an intestinal obstruction.

50. The Answer is B. The gastrocnemius can flex the knee joint and also plantar flex the foot. The tibialis posterior can plantar flex and invert the foot. The soleus can plantar flex the foot. The peroneus longus can plantar flex and evert the foot. The flexor digitorum longus can plantar flex the foot and flex the lateral four toes.

50. A 14-year-old gymnastic silver medalist falls from the parallel bar and complains of pains from the knee and ankle joints. On physical examination, her physician found that the muscle responsible for flexing the leg at the knee joint and plantar flexing the foot is severely weakened. Which of the following muscles involved in both movements was most likely damaged in this accident? (A) Tibialis posterior (B) Gastrocnemius (C) Soleus (D) Peroneus longus (E) Flexor digitorum longus

50. A 53-year-old woman with known kidney disease presents to a hospital because her pain has become increasingly more severe. A physician performing kidney surgery must remember that: (A) The left kidney lies a bit lower than the right one (B) The perirenal fat lies external to the renal fascia (C) The renal fascia does not surround the suprarenal gland (D) The left renal vein runs anterior to both the aorta and the left renal artery (E) The right renal artery is shorter than the left renal artery

50. The answer is D. The left renal vein runs anterior to both the aorta and the left renal artery. The renal fascia lies external to the perirenal fat and internal to the pararenal fat, and it also surrounds the suprarenal gland. The right renal artery runs behind the IVC and is longer than the left renal artery. Because of the large size of the right lobe of the liver, the right kidney lies a little lower than the left kidney.

50. A general surgeon is giving a lecture to a team of surgery residents. She describes characteristics of structures above the pectinate line of the anal canal, which include: (A) Stratifi ed squamous epithelium (B) Venous drainage into the caval system (C) Lymphatic drainage into the superfi cial inguinal nodes (D) Visceral sensory innervation (E) External hemorrhoids

50. The answer is D. The pectinate line is a point of demarcation between visceral and somatic portions of the anal canal. Characteristics above the pectinate line include columnar epithelium, venous drainage into the portal system, lymphatic drainage into the internal iliac nodes, visceral sensory innervation, and internal hemorrhoids.

51. The Answer is C. The semitendinosus extends the thigh and flexes the leg. The short head of the biceps femoris flexes the leg. The adductor magnus adducts, flexes, and extends the thigh. The sartorius and gracilis can flex the thigh and leg.

51. A 28-year-old basketball player falls while rebounding and is unable to run andjump. On physical examination, he has pain and weakness when extending his thigh and flexing his leg. Which muscle involved in both movements is most likely injured? (A) Short head of biceps femoris (B) Adductor magnus (C) Semitendinosus (D) Sartorius (E) Gracilis

51. A neonatal baby was born with diabetes mellitus due to an inadequate production of insulin. Cells in the endocrine portion of the pancreas that secrete insulin, glucagon, and somatostatin are derived from which of the following? (A) Ectoderm (B) Mesoderm (C) Endoderm (D) Proctodeum (E) Neural crest cells

51. Answer is C. Cells in the islets of Langerhans, an endocrine portion of the pancreas, are derived from the endoderm of the caudal foregut (from the liver diverticulum). Proctodeum is an invagination of the ectoderm of the terminal part of the hindgut.

51. A 5-month-old boy is admitted to the children's hospital because of urine being expelled from the dorsal aspect of the penis. Which of the following embryologic structures failed to fuse in this patient? (A) Labioscrotal swellings or folds (B) Urogenital sinus (C) Spongy urethra (D) Phallus (E) Urethral folds

51. The answer is C. A developmental defect in the spongy urethra results in epispadias, causing the patient to pass urine through an opening on the dorsum of the penis. Labioscrotal swellings form the scrotum in males and the labia majora in females. Urogenital sinus forms the urinary bladder, urethra, prostate, and bulbourethral glands in males, and the bladder, urethra, lower vagina, and greater vestibular glands in females. The phallus (genital tubercle) forms the penis in males and the clitoris in females. Urethral (urogenital) folds form the spongy urethra and a portion of the shaft of the penis in males and the labia minora in females.

52. During development, the midgut artery appears to be markedly narrowed at its origin. Which of the following structures is derived from the midgut and may receive inadequate blood supply? (A) Gallbladder (B) Stomach (C) Descending colon (D) Ascending colon (E) Rectum

52. Answer is D. The ascending colon is derived from the midgut. The gallbladder and stomach are derived from the foregut, and the descending colon and rectum are derived from the hindgut.

52. A 78-year-old man has carcinoma of the rectum. The cancer is likely to metastasize via the veins into which of the following structures? (A) Spleen (B) Kidney (C) Liver (D) Duodenum (E) Suprarenal gland

52. The answer is C. Cancer cells from rectal cancer are likely to metastasize to the liver via the superior rectal, inferior mesenteric, splenic, and portal veins. Cancer cells are not directly spread to the other organs listed. The spleen and duodenum drain their venous blood to the portal venous system, and the kidney and suprarenal gland empty into the caval (inferior vena cava) system

52. The Answer is D. The extensor hallucis longus can dorsiflex and invert the foot. The peroneus longus, peroneus tertius, and extensor digitorum longus can dorsiflex and evert foot. The peroneus brevis can plantar flex and evert the foot.

52. Which muscle can both dorsiflex and invert the foot? (A) Peroneus longus (B) Peroneus brevis (C) Peroneus tertius (D) Extensor hallucis longus (E) Extensor digitorum longus

53. A 3-year-old boy is admitted to the children's hospital with complaints of restlessness, abdominal pain, and fever. An MRI examination reveals that he has a double ureter. Which of the following embryonic structures is most likely failed to develop normally? (A) Mesonephric (Wolffian) duct (B) Paramesonephric (Müllerian) duct (C) Ureteric bud (D) Metanephros (E) Pronephros

53. Answer is C. The ureteric bud is an outgrowth of the mesonephric duct and develops into the ureter, renal pelvis, calyces, and collecting tubules. However, a bifurcated ureteric bud results in a partial duplication (bifid) of the ureter, whereas two separate ureteric buds result in a complete duplication. Mesonephric duct forms efferent ductules, epididymal duct, ductus deferens, ejaculatory duct, and seminal vesicles. Paramesonephric duct regress and its vestigial remnants form the appendix testis. Metanephros develops into the adult kidney. Pronephros degenerates and never forms functional nephrons.

53. The Answer is C. The quadriceps tendon is ruptured. The quadriceps muscle is a powerful knee extensor used in climbing, running, jumping, and rising from a seated position.

53. Rupture of the tendon superior to this structure would most likely cause an inability to extend the knee joint.

54. The Answer is A. The adductor tubercle is fractured. The adductor magnus inserts on the adductor tubercle on the femur and functions to adduct, flex, and extend the thigh.

54. Fracture of this structure would most likely cause weakness in adduction, flexion, and extension of the thigh.

54. A neonate has a small reducible protrusion through a defined ring at the umbilicus. His pediatrician indicates to the parents that this will likely close spontaneously. Which of the following congenital malformations is present? (A) Umbilical hernia (B) Symptomatic patent urachus (C) Patent omphalomesenteric duct (D) Omphalocele (E) Gastroschisis

54. The answer is A. In most case, an umbilical hernia closes spontaneously by age 4 and requires no surgery unless there is incarceration. A symptomatic patent urachus (drainage of urine at the umbilicus) is typically surgically excised. A patent omphalomesenteric duct (Meckel diverticulum) is promptly repaired to minimize the potential for intestinal obstruction or prolapse. Omphalocele and gastroschisis are defects that require surgical repair.

55. The Answer is B. The anterior tibial artery enters the anterior compartment by passing through a gap between the neck of the fibula and tibia. Therefore, a knife wound through the gap may cause an injury to the anterior tibial artery, resulting in muscle ischemia in the anterior compartment of the leg.

55. A knife penetrating through this point would most likely cause muscle ischemia in the anterior compartment of the leg.

56. The Answer is E. The common peroneal nerve is vulnerable to injury as it passes behind the head of the fibula and then winds laterally around the neck of the fibula. Fracture of the fibular head causes a lesion of the common peroneal nerve, resulting in paralysis of the muscles in the anterior and lateral compartments of the leg.

56. Fracture of this structure would most likely cause a lesion of the common peroneal nerve, resulting in paralysis of the muscles in the anterior and lateral compartments of the leg.

57. The Answer is D. The iliopsoas muscle is the chief flexor of the thigh and inserts on the lesser trochanter.

57. Which structure in this radiograph may be fractured, resulting in loss of the chief flexor of the thigh?

58. The Answer is B. The greater trochanter is the site for insertion of the obturator internus muscle tendon, which leaves the pelvis through the lesser sciatic foramen.

58. Fracture of which structure may destroy the site of insertion of the muscle that can rotate the thigh laterally and its tendon that passes through the lesser sciatic foramen?

59. The Answer is C. The ischiopubic ramus and ischial tuberosity provide attachment for the adductor magnus.

59. Which fractured structure is likely to cause paralysis of the adductor magnus?

6. The Answer is C. The popliteus muscle rotates the femur laterally ("unlocks" the knee) or rotates the tibia medially, depending on which bone is fixed. This action results in unlock- ing of the knee joint to initiate flexion of the leg at the joint. The rectus femoris flexes the thigh and extends the knee. The gastrocnemius flexes the knee and plantar flexes the foot. The semimembranosus extends the thigh and flexes and rotates the leg medially. The biceps femoris extends the thigh and flexes and rotates the leg laterally.

6. A 22-year-old patient is unable to "unlock" the knee joint to permit flexion of the leg. Which of the following muscles is most likely damaged? (A) Rectus femoris (B) Semimembranosus (C) Popliteus (D) Gastrocnemius (E) Biceps femoris

6. A 35-year-old woman comes to a local hospital with abdominal tenderness and acute pain. On examination, her physician observes that an abdominal infection has spread retroperitoneally. Which of the following structures is most likely affected? (A) Stomach (B) Transverse colon (C) Jejunum (D) Descending colon (E) Spleen

6. The answer is D. The descending colon is a retroperitoneal organ. The rest of the organs are surrounded by peritoneum.

256. The pronounced mediastinal shift to the right includes all of the following structures EXCEPT the: A. Sternum B. Aorta C. Trachea D. Esophagus E. Heart

A. Sternum

6. A 59-year-old woman comes to a local hospital for uterine cancer surgery. As the uterine artery passes from the internal iliac artery to the uterus, it crosses superior to which of the following structures that is sometimes mistakenly ligated during such surgery? (A) Ovarian artery (B) Ovarian ligament (C) Uterine tube (D) Ureter (E) Round ligament of the uterus

6. The answer is D. The ureter runs under the uterine artery near the cervix; thus, the ureter is sometimes mistakenly ligated during pelvic surgery. The other structures mentioned are not closely related to the uterine artery near the uterine cervix.

60. The Answer is A. The distal part of the femoral head receives blood mainly from the medial femoral circumflex artery, whereas the proximal part is supplied by a branch from the posterior division of the obturator artery.

60. Which structure becomes necrotic after the medial femoral circumflex artery is severed?

61. The Answer is A. The body of the talus has a groove on its posterior surface for the flexor hallucis longus tendon. This tendon also occupies the groove on the undersurface of the sustentaculum tali.

61. The flexor hallucis longus tendon is damaged in a groove on the posterior surface of a tarsal bone. Which bone in the radiograph is likely fractured?

62. The Answer is E. The first or medial cuneiform bone provides insertions for the tibialis anterior, tibialis posterior, and peroneus longus muscles.

62. The tibialis anterior and peroneus longus muscles are weakened. Which bone in the radiograph is most likely fractured?

63. The Answer is D. The spring (plantar calcaneonavicular) ligament extends from the sustentaculum tali of the calcaneus to the navicular bone.

63. The medial longitudinal arch of the foot is flattened because the spring ligament is torn. Which bone in the radiograph is most likely fractured?

64. The Answer is B. The cuboid bone has a groove for the peroneus longus muscle tendon.

64. The peroneus longus muscle tendon is damaged in a groove of a tarsal bone by fracture. Which bone in the radiograph is most likely fractured?

7. The Answer is C. The deep peroneal nerve supplies the anterior muscles of the leg, includ- ing the tibialis anterior, extensor hallucis longus, extensor digitorum longus, and peroneus tertius muscles, which dorsiflex the foot. The medial branch of the deep peroneal nerve supplies the skin of adjacent sides of the great and second toes, whereas the lateral branch supplies the extensor digitorum brevis and extensor hallucis brevis. The superficial pero- neal nerve innervates the peroneus longus and brevis, which plantar flexes the foot, and supplies the skin on the side of the lower leg and the dorsum of the ankle and foot. The tibial nerve innervates the muscles of the posterior compartment that plantar flexes and supplies the skin on the heel and plantar aspect of the foot. The lateral plantar nerve innervates muscles and skin of the lateral plantar aspect of the foot. The sural nerve sup- plies the skin on the posterolateral aspect of the leg and the lateral aspect of the foot and the little toe.

7. A patient presents with sensory loss on adjacent sides of the great and second toes and impaired dorsiflexion of the foot. These signs probably indicate damage to which of the following nerves? (A) Superficial peroneal (B) Lateral plantar (C) Deep peroneal (D) Sural (E) Tibial

7. During an annual health examination of a 46-year-old woman, a physician finds hypersecretion of norepinephrine from her suprarenal medulla. Which of the following types of nerve fibers are most likely overstimulated? (A) Preganglionic sympathetic fibers (B) Postganglionic sympathetic fibers (C) Somatic motor fibers (D) Postganglionic parasympathetic fibers (E) Preganglionic parasympathetic fibers

7. The answer is A. The suprarenal medulla is the only organ that receives preganglionic sympathetic fibers. No other nerve fibers are involved in secretion of norepinephrine from the suprarenal medulla.

7. A 29-year-old woman is admitted to a hospital because the birth of her child is several days overdue. Tearing of the pelvic diaphragm during childbirth leads to paralysis of which of the following muscles? (A) Piriformis (B) Sphincter urethrae (C) Obturator internus (D) Levator ani (E) Sphincter ani externus

7. The answer is D. The pelvic diaphragm is formed by the levator ani and coccygeus, whereas the urogenital diaphragm consists of the sphincter urethrae and deep transverse perinei muscles. The piriformis passes through the greater sciatic notch and inserts on the greater trochanter of the femur. The obturator internus forms the lateral wall of the ischiorectal fossa. The sphincter ani externus is composed of three layers, including the subcutaneous (corrugator cutis ani), superfi cial, and deep portions, and maintains a voluntary tonic contracture

8. The Answer is A. The superficial peroneal nerve supplies the peroneus longus and brevis muscles. Other muscles are innervated by the deep peroneal nerve.

8. A motorcyclist falls from his bike in an accident and gets a deep gash that severs the superficial peroneal nerve near its origin. Which of the following muscles is paralyzed? (A) Peroneus longus (B) Extensor hallucis longus (C) Extensor digitorum longus (D) Peroneus tertius (E) Extensor digitorum brevis

8. A 37-year-old small business manager receives a gunshot wound in the pelvic cavity, resulting in a lesion of the sacral splanchnic nerves. Which of the following nerve fi bers would primarily be damaged? (A) Postganglionic parasympathetic fi bers (B) Postganglionic sympathetic fi bers (C) Preganglionic sympathetic fi bers (D) Preganglionic parasympathetic fi bers (E) Postganglionic sympathetic and parasympathetic fi bers

8. The answer is C. The sacral splanchnic nerves consist primarily of preganglionic sympathetic neurons and also contain GVA fi bers. None of the other fi bers listed are contained in these nerves.

8. A 6-year-old girl comes to her pediatrician with constipation, abdominal distention, and vomiting. After thorough examination, she is diagnosed as having Hirschsprung disease (aganglionic megacolon), which is a congenital disease and leads to dilation of the colon. This condition is caused by an absence of which of the following kinds of neural cell bodies? (A) Sympathetic preganglionic neuron cell bodies (B) Sympathetic postganglionic neuron cell bodies (C) Parasympathetic preganglionic neuron cell bodies (D) Parasympathetic postganglionic neuron cell bodies (E) Sensory neuron cell bodies

8. The answer is D. Aganglionic megacolon (Hirschsprung disease) is caused by the absence of enteric ganglia (parasympathetic postganglionic neuron cell bodies) in the lower part of the colon, which leads to dilatation of the colon proximal to the inactive segment, resulting in an inability to evacuate the bowels. The other neuron cell bodies listed are not involved in this condition.

9. The Answer is D. To avoid damaging the sciatic nerve during an intramuscular injection, the clinician should insert the needle in the upper lateral quadrant of the gluteal region. The inserted needle in the lower medial quadrant may damage the pudendal and sciatic nerves. The inserted needle midway between the ischial tuberosity and the lesser trochanter may damage the sciatic and posterior femoral cutaneous nerves on the quadratus femoris. The inserted needle over the sacrospinous ligament may damage the pudendal nerve and vessels.

9. A 67-year-old patient has been given a course of antibiotics by gluteal intramuscu-lar injections after a major abdominal surgery. To avoid damaging the sciatic nerve during an injection, the needle should be inserted into which of the following areas? (A) Over the sacrospinous ligament (B) Midway between the ischial tuberosity and the lesser trochanter (C) Midpoint of the gemelli muscles (D) Upper lateral quadrant of the gluteal region (E) Lower medial quadrant of the gluteal region

9. A pediatric surgeon is resecting a possible malignant mass from the liver of a neonate with cerebral palsy. The surgeon divides the round ligament of the liver during surgery. A fibrous remnant of which of the following fetal vessels is severed? (A) Ductus venosus (B) Ductus arteriosus (C) Left umbilical vein (D) Right umbilical vein (E) Umbilical artery

9. The answer is C. The left umbilical vein becomes the round ligament of the liver after birth. The right umbilical vein did not leave a fibrous remnant because it was degenerated during the early embryonic period. The ductus venosus forms the ligamentum venosum; the ductus arteriosus forms the ligamentum arteriosum; the umbilical artery forms the medial umbilical ligament.

9. A young couple is having diffi culty conceiving a child. Their physician at a reproduction and fertility clinic explains to them that: (A) The ovary lies within the broad ligament (B) The glans clitoris is formed from the corpus spongiosum (C) Erection of the penis is a sympathetic response (D) Ejaculation follows parasympathetic stimulation (E) Fertilization occurs in the infundibulum or ampulla of the uterine tube

9. The answer is E. Fertilization takes place in the infundibulum or ampulla of the uterine tube. The glans clitoris is derived from the corpora cavernosa, whereas the glans penis is the expanded terminal part of the corpus spongiosum. Erection of the penis is caused by parasympathetic stimulation, whereas ejaculation is mediated via the sympathetic nerve. The ovaries are not enclosed in the broad ligament, but their anterior surface is attached to the posterior surface of the broad ligament

214. The midgut loop normally herniates through the primitive umbilical ring into the embryonic coelom during week 6 of development. Failure of the intestinal loop to return to the abdominal cavity by week 11 results in the formation of: A. Omphalocele B. Gastroschisis C. Anal agnesis D. Ileal diverticulum E. Intestinal stenosis

A. Omphalocele

210. The mesencephalic nucleus of the trigeminal nerve contains cells for the: A. Proprioception B. Pain C. Temperature D. Touch E. Pressure

A. Proprioception

173. Isolated damage to the flucculonodular lobe produces: A. Disturbances of the equilibrium B. Changes of the muscle tone C. Incoordinated movements D. Asynergia E. Decomposition of movements

A. Disturbances of the equilibrium

270. The saphenous nerve is a terminal branch of which of the following nerves? A. Femoral B. Obturator C. Genitofemoral D. Ilioinguinal E. Sciatic

A. Femoral

212. The middle ethmoid sinuses drain into the A. Middle meatus of the nose B. Superior meatus of the nose C. Sphenoethmoidal recess D. Inferior meatus of the nose E. Nasolacrimal duct

A. Middle meatus of the nose

246. A posteriorly perforating ulcer in the pyloric antrum of the stomach is most likely to produce initial localized peritonitis or abscess formation in the: A. Omental bursa B. Right subhepatic space C. Left subhepatic and hepatorenal spaces (pouch of Morison) D. Right subphrenic space E. Greater sac

A. Omental bursa

238. The parotid gland receives postganglionic parasympathetic fibers from the: A. Otic ganglion B. Submandibular ganglion C. Pterygopalatine ganglion D. Geniculate ganglion E. Superior cervical sympathetic ganglion

A. Otic ganglion

235. Parasympathetic fibers from CN IX synapse in which ganglion before traveling to the parotid gland? A. Otic ganglion B. Ciliary ganglion C. Submandibular ganglion D. Pterygopalatine ganglion E. Trigeminal ganglion

A. Otic ganglion

236. Parasympathetic fibers to the bladder are derived from which of the following nerves? A. Pelvic splanchnic B. Greater splanchnic C. T11-L2 D. Superior hypogastric plexus E. Sacral plexus

A. Pelvic splanchnic

252. The primary somatosensoru cortex is located in the: A. Postcentral gyrus B. Precentral gyrus C. Temporal gyrus D. Cuneus E. Superior temporal gyrus

A. Postcentral gyrus

272. The sensory tracts from the thalamus to the cerebral cortex pass through the: A. Posterior limb of the internal capsule B. Anterior limb of the internal capsule C. External capsule D. Extreme capsule E. Corpus callosum

A. Posterior limb of the internal capsule

220. Myocardial contraction normally is initiated at: A. The sinoatrial node B. The atrioventricular node C. The atrioventricular bundle D. Both atrioventricular node and bundle E. A localized region of ventricular myocardium

A. The sinoatrial node

267. A right-leg paralysis characterized as spastic may indicate damage to: A. The medial aspect of the left precentral gyrus B. The lateral aspect of the left precentral gyrus C. The lateral aspect of the right precentral gyrus D. The anterior horn cell E. The right fasciculus fracilis

A. The medial aspect of the left precentral gyrus

274. The sinoatrial node in the heart receives its blood supply principally from: A. The right coronary artery B. The posterior interventricular branch of the right coronary artery C. The anterior interventricular branch of the left coronary artery D. The circumflex branch of the left coronary arter E. None of the above

A. The right coronary artery

259. Ptosis results from a lesion of which nerve? A. Optic nerve B. Oculomotor nerve C. Trochlear nerve D. Trigeminal nerve E. Abducens nerve

B. Oculomotor nerve

255. The prominent muscle of the anterior wall of the axilla is: A. Pectoralis minor B. Pectoralis major C. Teres minor D. Subscapularis E. Intercostal

B. Pectoralis major

215. The most common interventricular septal defect (VSI) seen clinically is: A. Persistent truncus arteriousus B. Membranous VSO C. Common ventricle D. Foramen secundum defect E. Premature closure of foramen ovale

B. Membranous VSO

183. The lateral wall of each cavernous sinus contains the following nerves EXCEPT: A. Oculomotor B. Abducens C. Trochlear D. Ophthalmic nerve E. Maxillary nerve

B. Abducens

189. The main site of passage of cerebrospinal fluid into venous blood is through the: A. Pterygoid plexus B. Arachnoid villi C. Cerebral veins D. Parietal foramina E. Meningeal veins

B. Arachnoid villi

181. The lateral portion of the clavicle serves for attachment of the A. Triceps muscle B. Deltoid muscle C. Subscapular muscle D. Biceps brachii muscle E. Coracobrachialis muscle

B. Deltoid muscle

187. The lumbar plexus A. Is formed from the ventral rami of the first two lumbar nerves and part of the thrid lumbar nerve B. Gives rise to the obturator and femoral nerves C. Includes the sciatic nerve D. Has no connection with the sympathetic trunk E. Gives rise to the pudendal nerve

B. Gives rise to the obturator and femoral nerves

221. The nasolacrimal duct drains into the A. Lacrimal sac B. Inferior meatus of the nose C. Superior meatus of the nose D. Middle meatus of the nose E. Sphenoethmoidal recess

B. Inferior meatus of the nose

185. The long head of the triceps muscle originates from the A. Supraglenoid tubercle B. Infraglenoid tubercle C. Coronoid process D. Crest of the lesser tubercle E. Radial groove of the humerus

B. Infraglenoid tubercle

179. The last order neurons of the auditory tract are in the: A. Trapezoid body B. Medial geniculate body C. Superior colliculus D. Lateral lemniscus E. Lateral geniculate body

B. Medial geniculate body

224. The nucleus that gives rise to motor fibers that innervate muscles of larynx, pharynx, and palate is the: A. Hypoglossal nucleus B. Nucleus ambiguus C. Nucleus solitarius D. Dorsal motor nucleus of the vagus E. Edinger-Westphal nucleus

B. Nucleus ambiguus

280. The space between the opposed vocal folds and arytenoid cartilages is known as the A. Glottis B. Rima glottidis C. Vestibule D. Rima vestibule E. Piriform recess

B. Rima glottidis

216. The most superior vertebral spine that ordinarily is palpable is that of the: A. Eight cervical vertebra B. Seventh cervical vertebra C. First thoracic vertebra D. Seventh thoracic vertebra E. None of the above

B. Seventh cervical vertebra

191. Malignancies of the external genitalis and perineal abscesses result in enlargement first of the: A. External iliac lymph nodes B. Superficial inguinal lymph nodes C. Internal iliac lymph nodes D. Popliteal lymph nodes E. Lumbar lymph nodes

B. Superficial inguinal lymph nodes

278. The skin around the umbilicus on the anterior abdominal wall gains its sensory innervation from: A. T8 B. T10 C. T12 D. L1 E. L2

B. T10

258. The proximal row of carpal bones includes all the following carpal bones except which? A. The pisiform B. The capitate C. The lunate D. The triquetral E. The scaphoid

B. The capitate

244. The posterior fornix is the deepest part of which of the following structures? A. Urinary bladder B. Vagina C. Deep perineal space D. Urethra E. Uterus

B. Vagina

237. The parotid duct penetrates which of the following muscles? A. Masseter B. Medial pterygoid C. Buccinator D. Superior pharyngeal constrictor E. Levator anguli oris

C. Buccinator

227. The obturator nerve innervates: A. Psoas major muscle B. Sartorius muscle C. Obturator externus muscle D. Obturator internus muscle E. Biceps femoris muscle

C. Obturator externus muscle

177. The lacrimal gland receives parasympathetic innervation from the: A. Trigeminal ganglion B. Otic ganglion C. Pterygopalatine ganglion D. Nodose ganglion E. Ciliary ganglion

C. Pterygopalatine ganglion

206. The medial and lateral malleoli articulate with which of the following bones? A. Femur B. Calcaneus C. Talus D. Cuboid E. Fibula

C. Talus

233. The ovarian arteries arise from which of the following arteries? A. Superior gluteal B. Inferior gluteal C. Abdominal aorta D. Superior rectal E. Internal pudendal

C. Abdominal aorta

176. A laceration or contusion in the parotid region or a fracture of the temporal bone might damage which nerve? A. Trigeminal nerve B. Abducens nerve C. Facial nerve D. Glossopharyngeal nerve E. Vagus nerve

C. Facial nerve

184. The latissimus dorsi inserts into which of the following structures? A. Acromion B. Coracoid process C. Floor of the intertubercular groove of the humerus D. Supraglenoid tubercle E. Greater tubercle of the humerus

C. Floor of the intertubercular groove of the humerus

223. The nucleus ambiguus supplies axons to cranial nerves: A. IX, X, XI, and XII B. VII, IX, and XI C. IX, X D. VII, XI, XII E. VIII, IX, X, XI

C. IX, X

230. One component of the middle ear is the: A. Organ of corti B. Auricle C. Incus D. Seala vestibule E. Meatus

C. Incus

182. The lateral ventricle A. Is located in brainstem B. Communicated directly with fourth ventricle C. Is bounded inferiorly by the thalamus and caudate nucleus D. Does not communicate with third ventricle E. Contains fibers forming interthalamic adhesion

C. Is bounded inferiorly by the thalamus and caudate nucleus

243. The posterior cord of the brachia! plexus is described correctly by all the following statements EXCEPT: A. It is formed by union of all the posterior divisions of the plexus B. It gives off the upper subscapular nerve C. It gives off the median nerve D. It gives off the axillary nerve E. It has the radial nerve as a terminal branch

C. It gives off the median nerve

231. On examination of a patient with carpal tunnel syndrome may reveal all the following symptoms and signs except which? A. Atrophy of the muscles of the thenar eminence B. Weakness in opposition of the thumb C. Loss of skin sensation on the medial part of the palm D. Loss of skin sensation on the anterior surface of the index finger E. Normal skin sensation on the anterior surface of the little finger

C. Loss of skin sensation on the medial part of the palm

265. Ribs A. Always articulate with the vertebrae and sternum B. All articulate with the sternum by means of cartilages C. May be classified as true, false, or floating D. May be considered part of the appendicular skeleton E. Do not support any parts of the body

C. May be classified as true, false, or floating

226. The obturator nerve, a branch of the lumbag plexus, emerges from the psoas muscle on its A. Anterior surface B. Posterior surface C. Medial side D. Upper border E. Lateral side

C. Medial side

208. The median nerve is formed by parts of the: A. Lateral and posterior cord B. Posterior division of the upper and middle trunks C. Medial and lateral cords D. Posterior divisions of the middle and lower trunks E. Medial and posterior cords

C. Medial and lateral cords

276. The skeletal system develops from the: A. Ectoderm B. Endoderm C. Mesoderm D. Epiblast E. Hypoblast

C. Mesoderm

264. The renal papillae empty into which of the following structures? A. Renal vein B. Ureter C. Minor calyces D. Renal pyramid E. Renal column

C. Minor calyces

251. The primary motor area is located in the: A. Superior frontal gyrus B. Middle frontal gyrus C. Precental gyrus D. Postcentral gyrus E. Insula

C. Precental gyrus

248. Postganglionic parasympathetic fibers that innervate glands of the nasal cavity originate in the: A. Trigeminal ganglion B. Otic ganglion C. Pterygopalatine ganglion D. Submandibular ganglion E. Ciliary ganglion

C. Pterygopalatine ganglion

271. The second neuron in the pathway carrying pain and temperature in the anterolateral system A. Crosses to the opposite side within one segment rostrally B. Crosses randomly along the longitudinal axis of the spinal cord C. Remains ipsilateral the lenght of the spinal cord D. Crosses the midline within a small area of the medulla E. Is involved only with intersegmental reflexes

C. Remains ipsilateral the lenght of the spinal cord

268. The right lymphatic duct drains lymph from the A. Right lower limb B. Left upper limb C. Right side of the thoracic cavity D. Right part of the abdominal E. Left lower limb

C. Right side of the thoracic cavity

175. Knowledge of the lymphatic drainage of the breast is clinically important because of the high incidence of breast tumors. The major pathway of lymphatic drainage from mammary gland is along lymphatic channels that parallel A. Tributaries of the internal thoracic (mammary) vessels to the parasternal (internal thoracic) nodes B. Tributaries of the intercostal vessels to the parasternal nodes and posterior mediastinal nodes C. Tributaries of the axillary vessels to the axillary nodes D. Tributaries of the thoracoacromial vessels to the apical (subclavian) nodes E. Subcutaneous venous networks to the contralateral breast and to the abdominal wall

C. Tributaries of the axillary vessels to the axillary nodes

263. The relationship ("water passing under the bridge") is an especially important one for surgery of which of the followign arteries? A. Ovarian B. Testicular C. Uterine D. Vaginal E. Obturator

C. Uterine

217. Muscles participating in swallowing which originate from the styloid process are innervated by cranial nerves A. III, VII, X B. V, VII, XI C. VII, IX, XII D. V, IX, X

C. VII, IX, XII

247. The posterior mediastinum contains A. Phrenic nerves B. Thymus C. Main bronchi D. Esophagus E. None of the above

D. Esophagus

225. Numerous ducts that open into prostatic urethra has the: A. Seminal vesicles B. Great vestibular gland C. Bulbourethral gland D. Prostate gland E. Minor vestibular gland

D. Prostate gland

228. Of the bronchopulmonary segment in the superior lobe of the left lung, which two are combined to form the lingula? A. Apical and posterior B. Posterior and anterior C. Anterior and superior D. Superior and inferior E. Apical and inferior

D. Superior and inferior

250. The primary motor area is designed as area: A. 3 B. 1 C. 2 D. 4 E. 6

D. 4

234. The paleocerebellum is frequently referred to as the: A. Flocculonodular lobe B. Superior and inferior semilunar lobules C. Fastigial nucleus D. Anterior lobe E. Posterior lobe

D. Anterior lobe

211. The middle ear includes the: A. Tragus B. Anthelix C. Cochlea D. Auditory ossicles E. Membranous labyrinth

D. Auditory ossicles

229. The omohyoid, the sternocleidomastoid and the posterior belly of digastric muscle form the boundaries of which of the following triangle? A. Occipital B. Submandibular C. Submental D. Carotid E. Omoclavicular

D. Carotid

242. The pharynx terminates at the level of the: A. Hyoid bone B. 2nd cervical vertebra C. Thyroid cartilage D. Cricoid cartilage E. Jugular arch

D. Cricoid cartilage

188. Main function of the triceps at elbow joint is: A. Pronation B. Supination C. Flexion D. Extension E. Adduction

D. Extension

269. Saphenous nerve is a branch of the: A. Sciatic nerve B. Pudendal nerve C. Obturator nerve D. Femoral nerve E. Ilioinguinal nerve

D. Femoral nerve

207. The median nerve innervates A. Biceps brachii muscle B. Teres major muscle C. Supinator muscle D. Flexor carpi radialis muscle E. Anconeus muscle

D. Flexor carpi radialis muscle

249. Preganglionic sympathetic neurons reside in the spinal cord A. In the anterior horn B. In the posterior horn C. In the intermediolateral gray at the level of C4 D. In the intermediolateral gray at the vertebral levels of T1-L2 E. In the intermediolateral gray at the levels Ll-S3

D. In the intermediolateral gray at the vertebral levels of T1-L2

262. The quadrigeminal plate is part of the: A. Roof of the fourth ventricle B. Medulla C. Diencephalon D. Midbrain tectum E. Pontine tegmentum

D. Midbrain tectum

253. The profunda brachii artery A. Arises from the axillary artery B. Supplies the muscles of the hand C. Is the main branch of the radial artery D. Originates from the brachial artery E. Passes with the median nerve

D. Originates from the brachial artery

260. The pudendal nerve: A. Arises from the lumbar plexus B. Passes through the suprapyriform foramen C. Innervates the gluteus medius muscle D. Passes through the pudendal canal E. Supplies the skin of the inguinal region

D. Passes through the pudendal canal

279. The small saphenous vein empties into which of the following veins? A. Femoral B. Tibial C. Peroneal D. Popliteal E. Great Saphenous

D. Popliteal

218. The musculocutaneous nerve A. Originates from the medial cord of the brachial plexus B. Passes on the posterior surface of the humerus C. Innervates the skin of the medial side of the forearm D. Supplies the biceps brachii muscle E. Passes with the ulnar artery

D. Supplies the biceps brachii muscle

266. The right kidney is associated anteriorly with all of the following, EXCEPT the: A. Liver B. Right colic flexure C. Duodenum / part II D. Tail of the pancreas E. Coils of the ileum

D. Tail of the pancreas

261. The quadrangular space in the region of the shoulder transmits the following structures except which? A. The axillary nerve B. The posterior circumflex humeral artery C. The lymphatic vessels D. The radial nerve E. The posterior circumflex humeral vein

D. The radial nerve

240. The peritoneal cavity contains which of the following? A. Liver B. Pancreas C. Large intestine D. Kidney E. Peritoneal fluid

E. Peritoneal fluid

257. The prosencephalon differentiates into the: A. Myelencephalon B. Metencephalon C. Mesencephalon D. Rhombencephalon E. Telencephalon and diencephalon

E. Telencephalon and diencephalon

245. The posterior layer of the sheath of the rectus abdominal muscle below the arcuate line is formed by the: A. Linea alba B. Conjoint tendon C. Linea semilunaris D. Linea semicircularis E. Transversalis fasci

E. Transversalis fasci

209. The median nerve supplies A. Coracobrachialis and brachialis muscles B. Flexor carpi ulnaris muscles C. Triceps and anconeus muscles D. Hypothenar muscles E. Abductor pollicis brevis muscle

E. Abductor pollicis brevis muscle

241. The pharyngeal arches each have their own associated: A. Cartilaginous structures B. Cranial nerves C. Vascular components D. Muscular components E. All are correct

E. All are correct

170. The internal acoustic meatus in the skull A. Is located in the body of the sphenoid bone B. Is located in the mastoid bone C. Allows passage of the glossopharyngeal nerve D. Allows passage of the facial nerve only E. Allows passage of the vestibulocochlear nerve and the facial nerve

E. Allows passage of the vestibulocochlear nerve and the facial nerve

273. The short head of the biceps brachii muscle shares a common origin with: A. Brachialis B. Pectoralis major C. Brachioradialis D. Latissimus dorsi E. Coracobrachialis

E. Coracobrachialis

172. The internal ear consists of all the following structures EXCEPT: A. Cochlea B. Semicircular canals C. Utricle D. Saccule E. Epitympanic recess

E. Epitympanic recess

232. The orbicularis oculi is innervated by A. Optic nerve B. Oculomotor nerve C. Trochlear nerve D. Trigeminal nerve E. Facial nerve

E. Facial nerve

213. The middle pharyngeal constrictor arises from the: A. Pterygomandibular raphe B. Cricoid cartilage C. Thyroid cartilage D. Torus tubarius E. Hyoid bone

E. Hyoid bone

169. The intermaxillary segment forms via the fusion of the: A. Maxillary prominences B. Mandibular prominences C. Palatine shelves D. Lateral nasal prominences E. Medial nasal prominences

E. Medial nasal prominences

239. The peritoneal cavity contains: A. The stomach B. The pancreas C. The spleen D. The kidneys E. None of the above

E. None of the above

186. Loss of cutaneous sensation on the medial side of the leg could indicate injury to the A. Superficial peroneal nerve B. Deep peroneal enrve C. Sural nerve D. Tibial nerve E. Saphenous nerve

E. Saphenous nerve

219. The myelin sheath of the peripheral nerve is formed by: A. Fibroblasts B. Spongioblasts C. Macrophages D. Oligodendroglia E. Schwann cells

E. Schwann cells

178. Large involunatry movements of one side of the body, called hemiballismus, occur typically after damage to the: A. Caudate nucleus B. Pallidum C. Putamen D. Striatum E. Subthalmic nucleus

E. Subthalmic nucleus

277. The skeleton of the lower limb can be described correctly by all of the following EXCEPT A. The pelvic girdle is attached firmly to the vertebral column B. The bones that compose the pelvic girdle fuse together in development C. The two coxal bones are firmly articulated with each other at the midline D. The fibula does not enter into the knee joint E. The hip joint is identical in all respects to the shoulder joint

E. The hip joint is identical in all respects to the shoulder joint

180. The lateral part of the clavicle serves as attachment of the A. Triceps brachii muscle B. Biceps brachii muscle C. Coracobrachial muscle D. Subscapularis muscle E. Trapezius muscle

E. Trapezius muscle

174. The jugular foramen transmits all of the following EXCEPT: A. Glossopharyngeal nerve B. Vagus nerve C. Accessory nerve D. Inferior petrosal and sigmoid sinuses E. Vertebral artery

E. Vertebral artery

initiation of the impulse for contraction of the heart is accomplished by the

SA node

the lymphatic drainage of the penis is to which of the following lymph nodes

Sacral

the urogenital sinus contributes to all of the following structures except the

Vestibule

all of the following statements concerning the large intestine are true except

all parts of the large intestine are covered by peritoneum

all of the following structures are located in the superior mediastinum except the

arch of azygos vein

all of the following structures may be observed on the visceral surface of the liver except the

bare area

the tendons of which of the following muscles form the lateral margin of the popliteal fossa

biceps femoris

the gluteus medius and gluteus minimus muscles have simialr charateristics except

bith are strong flexor and medial rotators

all of the following structures are located in the posterior mediastinum except the

bronchi

which of the following structures is included in the female pudendum

bulb of the vestibule

the pelvic diaphragm includes which of the following muscles

coccygeus

posteriorly, the perineum ends at the

coccyx

which of the following arteries is a branch of the celiac trunk

common hepatic

portal-systemic anastomoses include all og the following except

communication of the superior and inferior mesenteric veins

the right atrium, select incorrect answer

forms the posterior surface of the heart

the inferior mesenteric artery

gives off the left colic artery

the superior mesenteric artery

gives off the right colic artery

the inferior gluteal nerve innervates the

gluteus maximus

the most powerful extensor of the thigh is the

gluteus maximus

which of the following structures is located in the anterior interventricular sulcus?

great cardiac vein

the sciatic nerve leaves the gluteal region for the thigh midway between the

greater trochanter and ischial tuberosity

each of the following structures enters or leaves the liver through the porta hepatis except

hepatic vein

the common bile duct passes in the

hepatoduodenal ligament

which ligament helps most to prevent overextension (hyperextension) at the hip joint

iliofemoral

all of the folllwing arteries are visceral branches of the internal iliac artery except

iliolumbar

all of the following arteries are visceral branches of the internal iliac artery except

iliolumbar

all the following arteries are branches of the internal iliac artery except the

iliolumbar

which muscle is a flexor of the hip joint

iliopsoas

what is true concerning the urinary bladder

in males its base lies on the prostate gland

all the following statements concering the uterus are correct except

in the erect posture, it lies directly on the superior surface of the bladder

meckels's diverticulum

is a remnant of the vitelline duct

in the male, the membranous urethra

is called the ejaculatory duct

each of the following statements about the great saphenous vein is correct except it

is found in the lateral crural compartment

right main bronchus

is more vertical than the left one

all of the following statements concerning the medial sural cutaneous nerve are correct except

it arises from the femoral

All of the following statements concerning the pudendal nerve are correct except

it arises in the plevis by roots from lumbar nerves 1, 3 and 4

all the following statements concering the pudenal nerver are correct except

it arises to most of the perineal skin surfaces

all of the following statements concerning the broad ligament are correct except

it assists in the formation of the transverse rectal folds

all the following statements concering the clitoris are correct except

it contains the female urethra

All of the following statements concerning the course of the femoral artery are correct EXCEPT:

it descends through the femoral canal

All of the following statements concerning the broad ligament of the uterus are correct except

it encloses the uterine tubes

all of the following statements concerning the lesser saphenous veins are correct except

it enters the leg in frint of the lateral malleolus

which of the following statements concerning the sacral plexus is correct?

it gives off the pudenal nerve

all of the following statements concerning the obturator artery are correct except

it is a branch of the external iliac artery

All of the following statements concerning the ilifemoral ligament are correct except

it is attached to the anterior superios iliac spine

All of the following statements concerning the perineum are correct except

it is bounded anterolaterally on either side by the superior ramus of the pubis

in the female the ureter has several relationships of special significance, including

it is crossed obliquely by the uterine artery

which of the following statements concerning the superficial inguinal ring is correct?

it is formed as a gap in the external oblique aponeurosis

considering the stomach

it is inervated by vagal nerves

All of the following statements concerning the talus are correct except

it is the largest and strongest bone of the foot

which of the statements correctly describes the obturator nerve

it passes along the lateral pelvic wall

all of the following statements concerning the rectum are correct except

it pierces the urogenital diaphragm

which of the following statements concerning the cervical dome of the pleura is correct

it projects through the superior thoracic aperture

which of the following statements correctly applies to the sigmoid colon

it recieves its blood supply from the left colic artery

which of the following statements concerning the ductus ejaculatorius is true?

it terminates in the prostatic part og the urethra

all the following statements concering the rectum are correct except

it usually has five folds in its interior

intermediate mesoderm will give rise to the

kidney

the superior vena cava returns blood from all these structures except the

lungs

which of the following structures is not located in the mediastinum

lungs

the adductor tubercle is located one the

medial epicondyle

which of the following structures is formed by remnants of the embryonic urachus

median ubilical fold

in spite of its rounded form the bladder is peaked anteriorly at its apex by the

median umbilical ligament

in spite of its rounded form, the bladder is peaked anteriorly at its apex by the

median umbilical ligament

the uteric bud grows out of the

mesonephric duct

the posterior interventricular sulcus lodges the

middle cardiac vein

the adductor of the hip joint are innervated by which of the following nerves

obturator

which of the following arteries is the chief blood supply of the perineum

obturator

the ureter. select the incorrect answer

on the left side crosses the inferior vena cava

which of the following structures is a retroperitoneal organ

pancreas

which structure is retroperitoneal

pancreas

which structure is retroperitoneal?

pancreas

the gonads develop from all of the following sources except

paramesonephric ducts

the pudendal nerve

passes through the pudendal canal

the pelvis is enclosed inferiorly by the

pelvic diaphragm

the parasympathetic preganglionic fibers to the pelvis comes form

pelvic splanchnic nerves

concerning the male urethra all of the following are true

penile part is compressed by the ischiovavernosus muscle

the ejaculatory duct opens into the

penis

muscles innervated by the deep peroneal nerve include all of the following except the

peroneus brevis

numerous ducts that open into prostatic urethra has the

prostate gland

the ejaculatory duct opens into the

prostatic urethra

which of the following nerves is the major innervation of the perineum

pudendal

the structure that passes through the lesser sciatic foramen is the

pudendal nerve

the patella is located in the tendon of which of the following muscles

quadriceps femoris

in a hospital emergency room, a 23-year old man states that he "inhaled" a peanut. on brochoscopy, the peanut wil likely be located in the

right lower lobar bronchus

All of the following statements regarding renal blood flow are correct except

right renal artery passes anterior to the inferior vena cava

each of the following veins belongs to the portal system except

right suprarenal vein

in the female, a strucutre which exits the pelvis to transverse the inguinal canal and terminates within the labia majora is the

round ligament of the uterus

each structure below lie in the broad ligament of the uterus except the

sacral part of the sympathetic trunk

which of the following nerves transverses the adductor canal

saphenous

the femoral nerve innervates the

sartorius muscle

select the incorrect statemnet. the lumbar plexus

sends a major branch which forms the pudenal nerve

each of the following muscles contributes directly to the stability of the knee joint except the

soleus muscle

the anterior surface og the liver lies against all of the following structures except

spleen

a direct branch of the celiac trunk is the

splenic artery

the arterial blood supply of the pancreas is from branches of the

splenic artery

which of the following structures is likely to be injured by a perforating gastric ulcer on the posterior wall of the stomach?

splenic artery

all the following veins belongs to the portal system except the

splenic vein

the trachea divides into the right and left bronchi at the level of which of the following structures

sternal angle

the anterior aspect of the kidney is in visceral relationship with the

stomach

the right kidney is associated anteriorly woth all of the following except the

tail of the pancreas

the lumbar plexus can be described correctly by which of the following statements

the 2nd, 3rd, and 4th lumbar nerves contribute to its branches

the kidney

the most posterior structure in the renal kindey is the renal artery

All of the following statements concerning the general structure of the testis are correct except

the rete testis coalesces to form the ejaculatory duct

select the incorrect statement

the right gastric artery is a branch of the celiac trunk

thein relation to vessels of the lower limb, which of the following statements is true

their arterial stem is the femoral artery

which of the following statements concerning the lateral femoral (gluteal) muscles is correct

they include strong abductors

all of the following statements concerning the poplitela space are correct except

this space is triangular

the medial plantar nerve is a branch of the

tibial nerve

a major muscle that dorsiflexes the foot is the

tibialis anterior

the posterior layer of the sheath of the rectus abdominis muscle is formed by the

transversalis fascia

taenia coli characterizes the

transverse colon

as the uterine artery passes from the anterior division of the internal iliac artery to the uterus, it crosses a structure that is sometimes mistakenly ligated during pelvic surgery. this structure is the

ureter

to the excretory part of the urinary system belong all of the following structures except the

ureters

the vesical trigone is bounded by which of the following structures at its inferior (anterior) angle

urethral aperture

the vesical trigone is bounded by wich of the following strucutres at its inferior (anterior) angle

urethral aperture

which organ can be reached without entering the peritoneal cavity

urinary bladder


Conjuntos de estudio relacionados

Chapter 6 - Business Strategy: Differentiation, Cost Leadership, and Blue Oceans

View Set

unit 2 Ribbons, Tabs, Help, and Status Bar

View Set

Chapter 7- Assessment of Health and Functioning

View Set